You are on page 1of 94

First Edition : May 2018

Published By :

Directorate of Studies

The Institute of Cost Accountants of India

CMA Bhawan, 12, Sudder Street, Kolkata – 700 016

www.icmai.in

Copyright of these study notes is reserved by the Institute of Cost Accountants


of India and prior permission from the Institute is necessary for reproduction of
the whole or any part thereof.
Work Book

STRATEGIC FINANCIAL MANAGEMENT

FINAL
GROUP – III
PAPER – 14

INDEX

Sl. No. Section – A : Investment Decisions Page No.

1 Investment Decisions, Project Planning and Control 1 – 16

2 Evaluation of Risky Proposals for Investment Decisions 17 – 27

3 Leasing Decisions 28 – 34

Section – B : Financial Markets and Institutions

4 Institutions in Finance Markets 35 – 36

5 Instruments in Financial Markets 37 – 47

6 Capital Markets 48 – 50

7 Commodity Exchange 51 – 53

Section – C : Security Analysis & Portfolio Management

8 Security Analysis & Portfolio Management 54 – 66

Section – D : Financial Risk Management

9 Financial Risks 67 - 69

10 Financial Derivatives – Instruments for Risk Management 70 – 79

11 Financial Risk Management in International Operations 80 - 88

Directorate of Studies, The Institute of Cost Accountants of India (Statutory Body under an Act of Parliament)
Work Book: Strategic Financial Management

Study Note – 1

Investment Decisions, Project Planning and Control

1. Choose the correct alternative:

(i) If the cost of an investment is ₹ 25000 and it results in a net cash inflow of `1800 per annum forever, the
Net Profitability Index of the investment is ___________(assume a discount rate of 8%)
a) 0.9
b) (-) 0.1
c) 1.11
d) 0.8

(ii) A project has the following cash flows:


Year 0 1 2 3
Cash Flow (` Lakh) -25 30 -15 40
If discount rate is 20%, then the NPV of the project is________
a) 11.75
b) 12.34
c) 12.74
d) 11.50

(iii) A project with an initial investment of ₹ 100 lakhs and life of 10 years generates cash flows after tax
(CFAT) of ₹ 20 lakh per annum. The Payback Reciprocal is ___________
a) 25%
b) 20%
c) 10%
d) 30%

(iv) The NPV of a 5 year project is ₹ 250 lakh and PVIFA at 10% for 5 years is 3.79. The Equivalent Annual
Benefit of the project is ___________
a) ` 65.96 lakh
b) ` 947.5lakh
c) ` 56.96 lakh
d) ` 96.65 lakh

(v) For an investment project, the following information is available.


Annual Cost Savings = ` 4,00,000; IRR = 15%; Useful life = 4 years; PVIFA (15%, 4) = 2.85.
The Payback Period is __________
a) 2.85 years
b) 2.89 years
c) 3.54 years
d) 2.95 years
Directorate of Studies, The Institute of Cost Accountants of India (Statutory Body under an Act of Parliament) Page 1
Work Book: Strategic Financial Management
(vi) The following information is available in case of an investment proposal:
NPV at discounting rate of 10% = ` 1250 and NPV at discounting rate of 11% = ` (-) 200. The IRR of the
proposal is_________________
a) 11.86%
b) 10.86%
c) 9.87%
d) 11.96%

(vii)The Profitability Index of a project is 1.28 and its cost of investment is ` 250000. The NPV of the project is
___________
a) ` 75,000
b) ` 80,000
c) ` 70,000
d) ` 65,000

(viii) From the following information calculate the MIRR of the project.
Initial Outlay ` 50000, cost of capital 12% p.a., Life of the project 4 years, Aggregate future value of cash
flows ` 104896.50.
a) 20.35%
b) 21.53%
c) 31.25%
d) 12.25%

Answer:

Question No. i ii iii iv v vi vii viii


Answer b c b a a b c a

2. An oil company proposes to install a pipeline for transport of crude from wells to refinery. Investments and
operating costs of the pipeline vary for different sizes of pipelines (diameter). The following details have
been conducted:
(a) Pipeline diameter (in inches) 3 4 5 6 7
(b) Investment required (` lakhs} 16 24 36 64 150
(c) Gross annual savings in operating costs before depreciation (` 5 8 15 30 50
lakhs)

The estimated life of the installation is 10 years. The oil company's tax rate is 50%. There is no salvage value
and straight line rate of depreciation is followed.

Calculate the net savings after tax and cash flow generation and recommend there from, the largest
pipeline to be installed, if the company desires a 15% post-tax return. Also indicate which pipeline will have
the shortest payback. The annuity PV factor at 15% for 10 years is 5.019.

Directorate of Studies, The Institute of Cost Accountants of India (Statutory Body under an Act of Parliament) Page 2
Work Book: Strategic Financial Management
Solution:

(1) Determination of CFAT (` lakhs)


Pipeline Diameter (inches) Gross Savings Depreciation Tax shield on Total cost
savings p.a. after tax depreciation savings (CFAT)
(2) 50% (4) X 50% (3) + (5)
(1) (2) (3) (4) (5) (6)
3 5 2.5 1.6 0.8 3.3
4 8 4.0 2.4 1.2 5.2
5 15 7.5 3.6 1.8 9.3
6 30 15.0 6.4 3.2 18.2
7 50 25.0 15.0 7.5 32.5
(2) Payback Period in years
Inches ` lakhs Years
3 16/3.3 4.848
4 24/5.2 4.615
5 36/9.3 3.871
6 64/18.2 3.516
7 150/32.5 4.615
Therefore, Pipeline diameter of 6 inches has shortest payback period.
(3) Determination of NPV (` lakhs)
Pipeline dia. CFAT PV factor @15% Total PV Cash NPV
(inches) for 10 years 10 Years Outflow
3 3.3 5.019 16.5627 16 0.5627
4 5.2 5.019 26.0988 24 2.0988
4 9.3 5.019 46.6767 36 10.6767
6 18.2 5.019 91.3458 64 27.3458
7 32.5 5.019 163.1175 150 13.1175

Suggestion - Pipeline of 6 inches diameter has highest NPV and it is recommended for installation.

3. Five Projects M, N, O, P and Q are available to a company for consideration. The investment required for
each project and the cash flows it yields are tabulated below. Projects N and Q are mutually exclusive.
Taking the cost of capital @ 10%, which combination of projects should be taken up for a total capital outlay
not exceeding `3 lakhs on the basis of NPV and Benefit-Cost Ratio (BCR)?
(`)
Project Investment Cash flow p.a. No. of years P.V. @10%
M 50,000 18,000 10 6.145
N 1,00,000 50,000 4 3.170
O 1,20,000 30,000 8 5.335
P 1,50,000 40,000 16 7.824
Q 2,00,000 30,000 25 9.077

Directorate of Studies, The Institute of Cost Accountants of India (Statutory Body under an Act of Parliament) Page 3
Work Book: Strategic Financial Management

Solution:

Total capital outlay < ` 3.00 lakh

Computation of Net Present Value and Benefit-Cost Ratio for five Projects
(`)
Project Investment Cash flow p.a. No. of P.V. @ P.V. NPV BCR
years. 10% (PV/Investment
M 50,000 18,000 10 6.145 1,10,610 60,160 2.212
N 1,00,000 50,000 4 3.170 1,58,500 58,500 1.585
O 1,20,000 30,000 8 5.335 1,60,050 40,050 1.334
P 1,50,000 40,000 16 7.824 3,12,960 1,62,960 2.086
Q 2,00,000 30,000 25 9.077 2,72,310 72,310 1.362
Statement Showing Feasible Combination of Projects and their NPV, BCR
Feasible combination of projects Investment (`) NPV (`) Rank BCR Rank
(i) M, N and P 3,00,000 2,82,070 1 1.940 1
(ii) M, N and O 2,70,000 1,59,160 4 1.589 4
(iii) O & P 2,70,000 2,03,010 3 1.752 3
(iv) M & Q 2,50,000 1,32,920 5 1.532 5
(v) N&P 2,50,000 2,21,460 2 1.886 2
(vi) N&Q 3,00.000 1,30,810 6 1.436 6

Comment - The optimum combination of projects, is Projects M, N and P with total investment of ` 3.00 lakhs
since it has highest NPV & BCR of ` 2,82,070 and 1.940 respectively. Hence, the same should be taken up.

4. S Ltd. has ` 10,00,000 allocated for capital budgeting purposes. The following proposals and associated
profitability indexes have been determined:

Project Amount (`) Profitability Index


1 3,00,000 1.22
2 1,50,000 0.95
3 3,50,000 1.20
4 4,50,000 1.18
5 2,00.000 1.20
6 4,00,000 1.05

Which of the above investments should be undertaken? Assume that projects are indivisible and there. Is no
alternative use of the money allocated for capital budgeting.

Directorate of Studies, The Institute of Cost Accountants of India (Statutory Body under an Act of Parliament) Page 4
Work Book: Strategic Financial Management
Solution:

Statement Showing Ranking NPV of Projects


Project Amount (`) Profitability Index PV of Cash inflow NPV
(1) (2) (3) (4) = (2)*(3) (5)=(4)-(2)
1 3,00,000 1.22 366000 66000
2 1,50,000 0.95 142500 (-)7500
3 3,50,000 1.20 420000 70000
4 4,50,000 1.18 531000 81000
5 2,00,000 1.20 240000 40000
6 4,00,000 1.05 420000 20000

Selection of projects: under NPV method (assuming the projects are indivisible and there is no alternative
use of unutilized amount), projects 3, 4 and 5 which will give a combined NPV of ` 191000 with no unutilized
amount, should be selected. (Detailed calculation of different alternative combinations must be given as
per the previous problem.)

5. GFM produces two products - a main product Cp and a co-product Dg. For their main product Cp there is a
100% buy back arrangement with their foreign collaborators. Recently GFM doubled their capacity and with
this their production capacity for the co-product Dg increased to 10,000 MT per annum. Fortunately, there
was an unprecedented increase in demand for Dg and price too has increased significantly to ` 1000 per
tonne.

However with delicensing and liberalisation, more and more units for manufacturing Cp and Dg are being
set up in the country. GFM, therefore, anticipates stiff competition for Dg from next financial year. For
maintaining sales at current level (i.e., 10,000 MT per year) GFM will have to drop the price by ` 50 per MT
every year for the next 5 years when prices are likely to stabilise at pre-boom level of ` 750 per MT.

The Vice-President (Marketing} who, sensing this situation, has just completed a market study, suggests that
the Company revive and earlier project for converting Dg into Dp grade and starting with 1,000 MT from next
year increase production of Dp in stages of 1,000 MT every year by correspondingly reducing Dg. The
Production Manger estimates that the additional variable cost for Dp will be ` 200 per MT. V.P. (Marketing)
feels that Dp can be sold at ` 1,500 per MT but in the first two years a discounted price of ` 1,400 in year 1
and `1,450 in year 2 will have to be fixed. With partial conversion into Dp, the drop in price of Dg can also be
contained at ` 25 MT instead of ` 50 envisaged. Production facilities for Dp involves a capital outlay of ` 50
lakhs.

Present the projected sales volume and price of products Dg and Dp for the next 5 years under two
alternatives.

If GEM normally appraises investments @ 12% p.a. and if cash beyond 5 years from investment are ignored,
advise whether Dp should be produced.

Directorate of Studies, The Institute of Cost Accountants of India (Statutory Body under an Act of Parliament) Page 5
Work Book: Strategic Financial Management
Solution:

Part I: Projected Sales Volume and Prices


Year Alternative I Alternative II
Dg qty. (MT) Price (`) Dg. qty. (MT) Price (`) Dp. qtv. (MT) Price (`)
1 10,000 950 9,000 975 1,000 1,400
2 10,000 900 8,000 950 2,000 1,450
3 10,000 850 7,000 925 3,000 1,500
4 10,000 800 6,000 900 4,000 1,500
5 10,000 750 5,000 875 5,000 1,500
Part II: For the revival of the earlier project for converting Dg partially into Dp the PV of the expected
additional contribution, if any, from Alternative II over that from Alternative I has to be considered.
Year wise Contributions
Year Incremental contribution from Dg Incremental contribution from Dp Total (` lakhs) (a+b)
(Qtv. MT) (`/MT) (` lakhs) (Qtv. MT)(a) (Rs/MT) (` lakhs)(b)
1 9,000 25* 2.25 1,000 250** 2.50 4.75
2 8,000 50 4.00 2,000 350 7.00 11.00
3 7,000 75 2.25 3,000 450 13.50 18.75
4 6,000 100 6.00 4,000 500 20.00 26.00
5 5,000 125 6.25 5,000 550 27.50 33.75
Working Notes:
* Incremental selling price = 975 - 950 = ` 25
** Incremental selling price - Incremental variable cost = (1,400 - 950) - 200 = ` 250 and so on.

Calculation of total PV
Year Incremental contribution (` lakhs) D.P. @12% Present value (` lakhs)
1 4.75 0.8929 4.24
2 11.00 0.7972 8.77
3 18.75 0.7118 13.35
4 26.00 0.6355 16.52
5 33.75 0.5674 19.15
62.03
NPV = 62.03 - 50 = ` 12.03 lakhs
Suggestion - Hence it will be advisable to start conversion of Dg into Dp proposed.

6. A particular project has a four-year life with yearly projected net profit of ` 10,000 after charging yearly
depreciation of ` 8,000 in order to write-off the capital cost of ` 32,000. Out of the capital cost ` 20,000 is
payable immediately (Year 0) arid balance in the next year (which will be the year 1 for evaluation). Stock
amounting to ` 6,000 (to be invested in year 0} will be required throughout the project and for debtors a
further sum of ` 8,000 will have to be invested in year 1. The working capital will be recouped in year 5. It is
expected that the machinery will fetch a residual value of ` 2,000 at the end of 4th year. Income tax is
payable @ 40% and the Depreciation equals the taxation writing down allowances of 25% per annum.

Directorate of Studies, The Institute of Cost Accountants of India (Statutory Body under an Act of Parliament) Page 6
Work Book: Strategic Financial Management
Income tax is paid after 9 months after the end of the year when profit is made. The residual value of ` 2,000
will also bear tax (a 40%. Although the project is for 4 years, for computation of tax and realisation of
working capital, the computation will be required up to 5 years.

Taking discount factor of 10%, calculate NPV of the project and give your comments regarding its
acceptability.

Solution:

Calculation of NPV of Project


Particulars Year
0 1 2 3 4 5
Capital expenditure (20,000) (12,000) - - - -
Working capital (6,000) (8,000 - - - -
Net profit - 10,000 10,000 10,000 10,000 -
Depreciation add back - 8,000 8,000 8,000 8,000 -
Tax - - (4,000) (4,000) (4,000) (4,800)
Salvage value - - - - 2,000 -
Recovery of working capital - - - - - 14,000
Net cash inflow (26,000) (2,000) 14,000 14,000 16,000 9,200
Discount factor (a 10% 1.000 0.9091 0.8264 0.7513 0.6830 0.6209
Present values (26,000) (1,818) 11,570 10,518 10,928 5,712

Suggestion - Since NPV is ` 10,910, it is suggested to accept the proposal.

7. T Ltd. has specialised in the manufacture of a particular type of transistors. Recently, it has developed a new
model and is confident of selling all the 8,000 units (new product) that would be manufactured in a year. The
required capital equipment would cost ` 25 lakhs and that would have an economic life of 4 years with no
significant salvage value at the end of such period. During the first four years, the promotional expenses
would be as planned below; (`)
Year 1 2 3 4
Expenses
Advertisement 1,00,000 75,000 60,000 30,000
Others 50,000 75,000 90,000 1,20,000

Variable costs of producing and selling a unit would be ` 250. Additional fixed operating costs to be
incurred because of this new product are budgeted at ` 75,000 per year. The management expects a
discounted return of 15% (after tax) on investments in the new product. You are required to work out an
initial selling price per unit of the new product that may be fixed with a view to obtaining the desired return
on investment. Assume a tax rate of 40"6 and use of straight line method of depreciation for tax purpose.

Note: The present value of annuity of ` 1 received or paid in a steady stream throughout the period of four
years in the future at 15% is 3.0079.

Directorate of Studies, The Institute of Cost Accountants of India (Statutory Body under an Act of Parliament) Page 7
Work Book: Strategic Financial Management

Solution:

Let the initial selling price per unit of new product be x


Total sales = 8,000 units  x = 8000x.V

Calculation of Cash Costs p.a. (`)


Variables costs (8,000 units X ` 250) 20,00,000
Advertisement and other expenses 1,50,000
Additional Fixed operating costs 75,000
Total cash costs p.a. 22,25,000

Depreciation p.a. = ` 25,00,000/4 years ` 6,25,000 p.a.


Profit before tax = 8,000* - (22,25,000 - 6,25,000) 8,000.x - 28,50,000
Tax ((& 40% on profit) = 0.40 (8,000* - 28,50,000) 3,200.x- 11,40,000
Total cash outflow = 22,25,000 +3,200.x- 11,40,000 3,200.r + 10,85,000
Net annual cash inflow = 8,000* - (3,200* + 10,85,000) 4,800.x- 10,85,000

Initial cash outflow = Present value of cash inflow


` 25,00,000 = (4,800.x- - 10,85,000) X 3.0079
25,00,000= 14,438.x - 32,63,571.50
14,438x = 25,00,000 + 32,63,571.50
14,438x = 57,63,571.50
x = 57,63,571.50/14,438 = ` 399.20

Hence, the initial selling price of new product is ` 399.20 per unit.

8. Modern Enterprises is considering the purchase of a new Computer System at a cost of ` 35 lakhs for its
Research and Development (R&D) Division. The cost of operation and maintenance (excluding
depreciation) will be ` 7 lakhs per annum. The useful life of the system will be 6 years after which it will have
a disposal value of ` 1 lakh. With the installation of the system there will be a reduction in running cost of ` 1
lakh per month in the R & D Division.

Moreover, the company is expected to receive ` 9 lakh immediately by disposal of some existing
equipments and furniture.

Capital expenditure in R & D will attract 100% write off for tax purpose. The effective tax rate of the company
may be taken as 50%. The gains arising from disposal of equipments and furniture are to be considered as
free of tax.

Taking the average cost of capital of the company as 12%, you are required to advise financial viability of
the proposal.

Directorate of Studies, The Institute of Cost Accountants of India (Statutory Body under an Act of Parliament) Page 8
Work Book: Strategic Financial Management
Solution:
(`)
Investment in 0th year: 35.00
Cost of new computer system 9.00
Less: Net realisation from disposal 26.00
Annual cash flow:
Annual saving in expense 12.00
Less: Maintenance cost 7.00
5.00
Net saving after tax @ 50% 2.50

Statement showing the present value of cash flow of the proposal (` lakhs)
Particulars Year Amount Factor Total present value
Investment 0 (26.00) 1.000 (26.00)
Tax saving 1 17.50 0.892 15.61
Annual 2.50 0.892 2.23
Net 2 2.50 0.797 1.99
Savings 3 2.50 0.711 1.78
4 2.50 0.635 1.59
5 2.50 0.567 1.42
6 2.50 0.506 1.27
Salvage 1.00 0.506 0.51
Net present value 0.40
The NPV being positive, the proposal is accepted.

9. A machine used on a production line must be replaced at least every four years. The costs incurred in
running the machine according to its age are:
(`)
Age of machine (years) 0 1 2 3 4
Purchase price 3,000
Maintenance 800 900 1,000 1,000
Repairs 200 400 800
Net Realisable value 1,600 1,200 800 400

Future replacement will be identical machines with the same costs. Revenue is unaffected by the age of the
machine. Assume there is no inflation and ignore tax. The cost of capital is 15%. Determine the optimum
replacement cycle.

Year 1 2 3 4
P.V. factors @ 15% 0.8696 0.7561 0.6575 0.5718
P.V. of annuity @ 15% 0.8696 1.6257 2.2832 2.8550

Directorate of Studies, The Institute of Cost Accountants of India (Statutory Body under an Act of Parliament) Page 9
Work Book: Strategic Financial Management
Solution:

The possible replacement options of the machine are every one, two, three and four years. The annual
equivalent cost of each of these replacement policies are as follows:

Replacement Every Year


Year 0 1
Cost (3,000) -
Maintenance - (800)
Resale value - 1600
Total (3000) 800
DCF @ 15% 1.0 0.8696
Present value of cash flows (3000) 696
Total present value of costs = ` 2,304
Annual equivalent cost = ` 2,304/0.8696 = ` 2,649

Replacement Every Two Years


Year 0 1 2
Cost (3000) - -
Maintenance - (800) (900)
Repairs - - (200)
Resale value - - 1200
Total (3000) (800) 100
DCF @ 15% 1.0 0.8696 0.7561
Present value of cash flows (3000) (696) 76
Total present value of costs = ` 3,620
Annual equivalent cost = ` 3,620/1.6257 = ` 2,227

Replacement every three years (`)


Years 0 1 2 3
Cost (3,000) • - -
Maintenance - (800) (900) (1,000)
Repairs - - (200) (400)
Net realisable value - - - 800
Total (3,000) (800) (1,100) (600)
DCF @ 15% 1.000 0.8696 0.7561 0.6575
Present value of cash flows (3,000) (696) (832) (395)

Total present value of costs = ` 4,923


Annual equivalent cost = ` 4,923/2.2832 = ` 2,156

Directorate of Studies, The Institute of Cost Accountants of India (Statutory Body under an Act of Parliament) Page 10
Work Book: Strategic Financial Management

Replacement every four years (`)


Years 0 1 2 3
Cost (3,000) - - - -
Maintenance - (800) (900) (1,000) (1000)
Repairs - - (200) (400) (800)
Net realisable value - - - - 400
Total (3,000) (800) (1,100) (1400) (1400)
DCF @ 15% 1.000 0.8696 0.7561 0.6575 0.5718
Present value of cash flows (3,000) (696) (832) (921) (800)

Total present value of costs = ` 6,249


Annual equivalent cost = ` 6,249/2.8550 = ` 2,189

Suggestion – Since, annual equivalent cost is the minimum in three years, the machine is suggested to be
replaced every three years.

10. Company has to replace one of its machines which has become unserviceable. Two options are available:
(i) A more expensive machine (EM) with 12 years of life, (0) A less expensive machine (LM) with 6 years of
life.

If machine LM is chosen, it will be replaced at the end of 6 years by another LM machine. The pattern of
maintenance, running costs and prices are as under:
(`)
Particulars EM LM
Purchase price 10,00,000 7,00,000
Scrap value at the end of life 1,50,000 1,50,000
Overhauling is due at the end of 8th year 4th year
Overhauling costs 2,00,000 1,00,000
Annual repairs 1,00,000 1,40,000
Cost of capital - 14%

You are required to recommend with supporting calculations which of the machines should be purchased.
End of 4th year 0.5921
End of 6th year 0.4556
End of 8th year 0.3506
End of 12th year 0.2076
Years 1 to 6 3.8890
Years 1 to 12 5.6600

Directorate of Studies, The Institute of Cost Accountants of India (Statutory Body under an Act of Parliament) Page 11
Work Book: Strategic Financial Management
Solution:

Machine EM - 12 years life


Particulars Year Cost (`) Discount factor Present value (`)
Purchase price 0 10,00,000 1.0000 10,00,000
Overhauling 8 2,00,000 0.3506 70,120
Annual repairs 1 – 12 1,00,000 5.6600 5,66,000
Scrap value 12 1,50,000 0.2076 (31,140)
Total NPV of outflow 16,04,980

Machine LM-6 years life


Particulars Year Cost (`) Discount factor Present value (`)
Purchase price 0 7,00,000 1.0000 7,00,000
Overhauling 4 1,00,000 0.5921 59,210
Annual repairs 6 1,40,000 3.8890 5,44,460
Scrap value 6 1,50,000 0.4556 (68,340)
Total NPV of outflow 12,35,330
Annualized value
EM = ` 16,04,980/5.660 = ` 2,83,565 LM = ` 12,35,330/3.889= ` 3,17,647
Since annualised value is less for more expensive machine, it is suggested to replace existing machine with
machine EM.

11. A company is considering a cost saving project. This involves purchasing a machine costing ` 7,000, which
will result in annual savings on wage costs of ` 1,000 and on material costs of ` 400.
The following forecasts are made of the rates of inflation each year for the next 5 years:
Wages costs 10%, Material costs 5%, General prices 6%
The cost of capital of the company, in monetary terms, is 15%.
Evaluate the project, assuming that the machine has a life of 5 years and no scrap value.

Solution:

Calculation of Net Present Value


Year Labour cost savings (`) Material Costs Savings (`) Total savings (`) DCF Present values @ 15% (`)
1 1000 X (1.1)= 1,100 400 X (l.05) = 420 1,520 0.870 1,322
2 1000 X (l.l)2= 1,210 400 X (l.05)2 = 441 1,651 0.756 1,255
3 1000 X (1.1)3= 1,331 400 X (l.05)3 = 463 1,794 0.658 1,184
4 1000X(1.1)4= 1,464 400 X (l.05)4 = 486 1,950 0.572 1,112
5 1000X(1.1)5= 1,610 400 X (l.05)5 = 510 2,120 0.497 1,060
Present value of total savings 5,933
Less. Initial cash outflow 7,000
NPV (-) 1,067

Directorate of Studies, The Institute of Cost Accountants of India (Statutory Body under an Act of Parliament) Page 12
Work Book: Strategic Financial Management
Suggestion: Since the present value of cost of project exceeds the cost of savings from it and hence it is not
suggested to purchase the machine.

12. ABC Enterprises Ltd. is evaluating an option to computerise their distribution system. The total capital cost for
the system is `100 lakhs. The operation and maintenance costs (excluding depreciation) per annum is
expected to be ` 10 lakhs. The computer system is expected to have an useful life of 5 years after which it is
expected to become obsolete and would require replacement. It would have negligible salvage value at
that time. The depreciation rate is 10 per cent on written down value method. There would a cost savings of
`10 lakhs due to reduction in clerical numbers, `20 lakhs due to space released and `10 lakhs on account
of inventory reduction. Previous trends indicate that costs are inflating at 10% per annum. The tax rate for the
firm is 5096. Advice whether the company should invest in the Computer system.

Solution:

Calculation of Depreciation (` lakhs)


Year 1 2 3 4 5
WDV at the beginning 100 90 81 72.9 65.61
Less. Depreciation 10 9 8.1 7.29 6.561
WDV at the end 90 81 72.9 65.61 59.049

Calculation of NPV (` lakhs)


Year 1 2 3 4 5
Savings:
Reduction in labour cost 10.00 11.00 12.10 13.31 14.64
Reduction in space 20.00 22.00 24.20 26.62 29.28
Savings in inventory 10.00 11.00 12.10 13.31 14.64
40.00 44.00 48.40 53.24 58.56
Costs: -
Operation and maintenance 10.00 11.00 12.10 13.31 14.64
Depreciation 10.00 9.00 8.10 7.29 6.56
20.00 20.00 20.20 20.60 21.20
Net savings 20.00 24.00 28.20 32.64 37.36
Less : Tax @50% 10.00 12.00 14.10 16.32 18.68
10.00 12.00 14.10 16.32 18.68
Add: Depreciation 10.00 9.00
Cash inflow after tax 20.00 21.00 22.20 23.61 25.24
Discount factor 0.909 0.826 0.751 0.683 0.621
Present values 18.18 17.35

Total present value of cash inflows = ` 84 lakhs


NPV = ` 84 lakhs - ` 100 lakhs = (-) `16
Analysis - Since NPV is negative, it is suggested not to invest in computerisation of distribution system.

Directorate of Studies, The Institute of Cost Accountants of India (Statutory Body under an Act of Parliament) Page 13
Work Book: Strategic Financial Management
13. Apex Enterprises is interested in assessing the cash flows associated with the replacement of the old
machine by a new machine. The old machine has a book value of ` 90,000 which can be sold for the same
amount. It has a remaining life of 5 years, after which the salvage value is expected to be 'nil'. It is being
depreciated annually @ 10% using the written down value method.

The new machine costs ` 4 lakhs, and has a resale value of ` 2.5 lakhs at the end of 5 years. The new
machine is expected to save manufacturing costs of ` 1 lakh p,a. Investment in working capital remains
same. The tax rate applicable to the firm is 50%.

You, as a Project Analyst, are required to work out the incremental cashflows associated with the
replacement of the old machine and to prepare a statement to be presented to the management for
consideration.

Solution:

Cash outflow (` lakhs)


Cost of the machine (new) 4.00
Less: Sale value of old machine (0.90)
3.10

Incremental Depreciation (` lakhs)


Year WDV Depreciation (@ 10%)
1 3.1000 0.3100
2 2.7900 0.2790
3 2.5110 0.2511
4 2.2599 0.2260
5 2.0339 0.2034

Statement Showing Incremental Cash flows and CFAT associated with Replacement of Old Machine with a
New Machine (` lakhs)
Particulars Year 1 Year 2 Year 3 Year 4 Total
Savings in manufacturing cost 1.0000 1.0000 1.0000 1.0000 1.0000
Less: Incremental depreciation 0.3100 0.2790 0.2511 0.2260 0.2034
Incremental taxable income 0.6900 0.7210 0.7489 0.7740 0.7966
Less: Tax @ 50% 0.3450 0.3605 0.3744 0.3870 0.3983
Incremental earning after tax (EAT) 0.3450 0.3605 0.3745 0.3870 0.3983
CFAT (EAT + Depreciation) 0.6550 0.6395 0.6256 0.6130 0.6017
Add : Salvage value - - - - 2.5000
Total Incremental CFAT 0.6550 0.6395 0.6256 0.6130 3.1017 5.6348
Less : Cash outflows 3.1000
Incremental net cash flows 2.5348

Suggestion - In view of positive incremental net cash flows, it is suggested to replace the existing machine.
Directorate of Studies, The Institute of Cost Accountants of India (Statutory Body under an Act of Parliament) Page 14
Work Book: Strategic Financial Management

14. The present output details of manufacturing department of X Ltd. are as follows:

Average output per week 48,000 units


Saleable value of output ` 60,000
Contribution made by above ` 24,000

The management plan to introduce more mechanisation in the department at a capital cost of `16,000. As
an effect of this the number of employees will be reduced from the .existing strength of 160 nos. to 120 nos.
but the output of individual employee will increase by 60%. As an incentive to achieve the extra output, the
management propose to offer an one per cent increase in the existing piece work price of Re. 0.10 per
article for every 2% increase in the individual output achieved, hi order to sell the increased output, it will be
necessary to reduce the sale price by 4%.

You are required to calculate extra weekly contribution resulting from the proposed changes, as above,
and give your recommendation.

Solution:

Current output per employee per week 48,000 units /160 Nos. = 300 units
Output per employee per week after mechanisation 300 units X 160/100 = 480 units
Total production after mechanisation 480 units X 120 employees = 57,600 units
Current selling price ` 60,000/48,000 units - ` 1.25
Revised selling price ` 1.25 X 96/100 = ` 1.20

Calculation of Revised Piece Rate Wages

Current rate ` 0.10


Incentive @ ` 0.05 p.u. for 60% increase in output 0.03
0.13

Calculation of Variable Cost Per Unit (`)


Sales 60000
Less: Contribution 24000
Variable cost including wages 36000
Less: Wages (48,000 units X ` 0.10) 4800
Variable cost excluding wages 31200

Variable cost {excluding wages) p.u. = ` 31,200/48,000 units = ` 0.65 p.u.

Directorate of Studies, The Institute of Cost Accountants of India (Statutory Body under an Act of Parliament) Page 15
Work Book: Strategic Financial Management
Revised Profitability Statement After Mechanisation (`)
Sales (57,600 units @ ` 1.20) 69120
Less: Variable cost
Wages @ ` 0.13 7488
Other Expenses @ ` 0.65 37440
44928
Contribution 24192
Current contribution 24000
Increase in contribution per week 192

Payback Period= Additional Investment/Incremental contribution per week = ` 16,000/`192 = 83.3 weeks or
1.6 years.

Directorate of Studies, The Institute of Cost Accountants of India (Statutory Body under an Act of Parliament) Page 16
Work Book: Strategic Financial Management

Study Note – 2

Evaluation of Risky Proposals for Investment Decisions

1. Choose the correct alternative:

(i) If the cash flows over the life of the project are perfectly correlated, the Standard Deviation is
determined using the formula ________
𝛔𝟐
a) SD =
(𝟏+𝐢)𝟐
𝛔
b) SD =
(𝟏+𝐢)𝟐
𝛔𝟐
c) SD =
(𝐢+𝐢)
𝛔𝐭
d) SD =
(𝟏+𝐢)𝐭

(ii) If nominal discounting rate is 15%, inflation rate is 5% , then real discounting rate will be ___________
a) 9.52%
b) 9.25%
c) 10.25%
d) 10.52%

(iii) If project cost = ` 12,000, Annual cash flow = ` 4,500 Cost of capital = 14% , life = 4 years, PVIFA (14%, 4)
= 2.9137, then the sensitivity with respect to the project cost is
a) 9.27%
b) 10.27%
c) 9.72%
d) 10.72%

(iv) The following information is available with respect to Project X


NPV Estimate (`) 30000 60000 120000 150000
Probability 0.1 0.4 0.4 0.1

The expected NPV will be _____________


a) ` 100000
b) `75000
c) `90000
d) `120000

(v) If expected NPV =` 120000 and S.D = `30000, then coefficient of variation will be _____________
a) 25%
b) 20%
c) 30%
d) 50%

Directorate of Studies, The Institute of Cost Accountants of India (Statutory Body under an Act of Parliament) Page 17
Work Book: Strategic Financial Management
(vi) Given, expected value of profit without perfect information = `1600 and expected value of perfect
information = ` 300, then expected value of profit with perfect information will be ________
a) `1300
b) `1900
c) `950
d) None of the above

Solution:

Question No. i ii iii iv v vi


Answer d a a c a b

2. A company has estimated the following demand level of its product:


Sales volume (units) 10000 12000 14000 16000 18000
Probability 0.10 0.15 0.25 0.30 0.20

It has assumed that the sales price of `6 per unit, marginal cost `3.50 per unit, and fixed costs ` 34,000. What
is the probability that: (a) the company will break-even in the period? (b) the company will make a profit of
at least ` 10,000?

Solution:

To break-even, the company must earn enough total contribution to cover its fixed costs. The contribution
to fixed costs and profit is `2.50 per unit (i.e. 6 - 3.5).

To break-even, sales must be as follows:


Contribution required/ Contribution per unit = ` 34,000/` 2.50 = 13600 units

The probability that sales will equal or exceed 13,600 units is the probability that sales will be 14,000, 16,000 or
18,000 units, which is (0.25 -f 0.30 + 0.20) = 0.75 or 75%

To earn profit of `10000, the company must earn enough contribution to cover its fixed costs (` 34,000) and
then make the profit, so total contribution must be ` 44,000. To earn this contribution, sales must be as
follows:
` 44,000/2.50 = 17,600 units

The probability that sales will equal or exceed 17,600 units is the probability of sales being 18,000 units, which
is 0.20 or 20%.

3. A company has estimated the unit variable cost of a Product to be `10, and the selling price is `15 per unit.
Budgeted sales for the year are 20,000 units. Estimated fixed costs are as follows:
Fixed costs p.a. (`) 50,000 60,000 70,000 80,000 90,000
Probability 0.1 0.3 0.3 0.2 0.1

Directorate of Studies, The Institute of Cost Accountants of India (Statutory Body under an Act of Parliament) Page 18
Work Book: Strategic Financial Management
What is the probability that the company will equal or exceed its target profit of `25,000 for the year?

Solution:

The different outcomes for fixed cost are mutually exclusive events. If fixed costs are ` 50,000 for example,
they can't be anything else as well.

Budgeted sales = 20,000 units


Budgeted unit contribution = 15 - 10 = ` 5

Budgeted total contribution (20,000X5) 100000


Target profit 25000
Maximum fixed costs if target is to be achieved 75000

The probability that fixed costs will be `75000 or less is:


= P (50,000 or 60,000 or 70,000)
= P (50,000) + P (60,000) + P (70,000)
= 0.1+0.3 +0.3
= 0.7 or 70%

4. The following table presents the proposed cash flows for projects M and N with their associated probabilities.
Which project has a higher preference for acceptance?
Project M Project N
Possibilities Cash flow (` lakhs) Probability (` lakhs) Cash flow (` lakhs) Probability (` lakhs)
1 7,000 0.10 12,000 0.10
2 8,000 0.20 8,000 0.10
3 9,000 0.30 6,000 0.10
4 10,000 0.20 4,000 0.20
5 11,000 0.20 2,000 0.50

Solution:

Calculation of Expected Value of Cash flow (` lakhs)


Project M Project N
Possibilities Cash flow Probability Expected value Cash flow Probability Expected value
1 7,000 0.1 700 12,000 0.10 1,200
2 8,000 0.2 1,600 8,000 0.10 800
3 9,000 0.3 2,700 6,000 0.10 600
4 10,000 0.2 2,000 4,000 0.20 800
5 11,000 0.2 2,200 2,000 0.50 1,000
1.0 EV = 9200 1.00 EV = 4400

Directorate of Studies, The Institute of Cost Accountants of India (Statutory Body under an Act of Parliament) Page 19
Work Book: Strategic Financial Management
Analysis - The expected monetary value of Project M is greater than Project N. Therefore, Project M has a
higher preference for acceptance.

5. The Managing Director of Y Ltd. has evolved some decision making to the operating division of the firm. He
is anxious to extend this process but first wishes to be assured that decisions are being taken properly in
accordance with group policy.

As a check on existing practice, he has asked for an investigation to be made into a recent decision to
increase the price of the sole product of Z division to ` 14.50 per unit but to rising costs.

The following information and estimates were available for the management of Z division:
Last year 75,000 units were sold at` 12 each with total units cost of ` 9 of which ` 6 were variable costs.

For the year ahead the following cost and demand estimates have been made:
Pessimistic Probability 0.15 ` 7.00 per unit
Most likely Probability 0.65 ` 6.50 per unit
Optimistic Probability 0.20 ` 6.20 per unit

Total fixed costs:


Pessimistic Probability 0.3 Increase by 50%
Most likely Probability 0.5 Increase by 25%
Optimistic Probability 0.2 Increase by 10%
Demand estimates at various prices (units) (Price per unit)
Particulars Probability ` 13.50 ` 14.50
Pessimistic 0.3 45,000 35,000
Most likely 0.5 60,000 55,000
Optimistic 0.2 70,000 68,000
(Unit variable costs, fixed costs and demand estimates are statistically independent)

For this type of decision the group has decided that the option should be chosen which has the highest
expected outcome with at least an 80% chance of breaking even.
You are required:
(a) to assess whether the decision was made in accordance with group guidelines,
(b) to obtain what is the group attitude to risk as evidenced by the guidelines.

Solution:

Demand Probability Contribution Probability Total Joint Cumulative joint


per unit contribution Probability Probability
Selling Price `13.50
45,000 0.3 6.50 0.15 2,92,500 0.045 0.045
7.00 0.65 3,15,000 0.195 0.240
7.30 0.20 3,28,500 0.060 0.300

Directorate of Studies, The Institute of Cost Accountants of India (Statutory Body under an Act of Parliament) Page 20
Work Book: Strategic Financial Management
60,000 0.5 6.50 0.15 3,90,000 0.075 0.375
7.00 0.65 4,20,000 0.325 0.700
7.30 0.20 4,38,000 0.100 0.800
70,000 0.2 6.50 0.15 4,55,000 0.030 0.830
7.00 0.65 4,90,000 0.130 0.960
7.30 0.20 5,11,000 0.040 1.000
Selling Price ` 14.50
35,000 0.3 7.50 0.15 2,62,500 0.045 0.045
8.00 0.65 2,80,000 0.195 0.240
8.30 0.20 2,90,500 0.060 0.300
55,000 0.5 7.50 0.15 4,12,500 0.075 0.375
8.00 0.65 4,40,000 0.325 0.700
8.30 0.20 4,56,500 0.100 0.800
68,000 0.2 7.50 0.15 5,10,000 0.030 0.830
8.00 0.65 5,44,000 0.130 0.960
8.30 0.20 5,64,400 0.040 1.000
Last year's fixed costs = 75,000 units X ` 3 =` 2,25,000
Estimated Fixed Costs (`)
` 2,25,000 X 1.10X0.2 49,500
` 2,25,000 X 1.25 X 0.5 1,40,625
` 2,25,000 X 1.50 X 0.3 1,01,250
2,91,375

To break-even the contribution must be greater than ` 291375. It is noticed from the above tables iat at
selling price of `13.50 there is 100% chance to break-even. However, at selling price of ` 14.50 there is 70%
chance of break-even. The selling price of ` 14.50, therefore, contravenes group guidelines.

Attitude to Risk - The group seeks to minimise the downside risk whilst maximising its return. It is to some extent
risk averse, but it is prepared to take some risk i.e., 20% risk of loss. It is always sought maximise its returns,
ignoring the probability of failure, it would be risk neutral.

6. (a) A Ltd. has a choice between three projects X, Y and Z. The following information has been estimated:
Projects Profit (` ‘000)
D1 D2 D3
X 190 50 15
Y 110 200 160
Z 150 140 110

Probabilities are D1 = 0.6, D2 = 0.2, D3 = 0.2


Which projects should be undertaken if decision is made by expected value approach?

(b) Calculate the expected value of perfect information?

Directorate of Studies, The Institute of Cost Accountants of India (Statutory Body under an Act of Parliament) Page 21
Work Book: Strategic Financial Management

Solution:

Calculation of Expected Values


Particulars Profit (` ‗000) Probability Profit *Probability
Project X D1 190 0.6 114
D2 50 0.2 10
D3 15 0.2 3
EV = 127
Project Y D1 110 0.6 66
D2 200 0.2 40
D3 160 0.2 32
EV=138
Project Z D1 150 0.6 90
D 140 0.2 28
D3 110 0.2 22
EV=140
Analysis - Project Z should be chosen because it has the highest EV of ` 140000.

(b) Perfect Information

In order to obtain perfect information about future states of demand from market researchers, a company
has to pay for the information. The maximum value of this perfect information will be equal the EV with the
information less the EV without information.

Demand Choose Profit (`'000) Probability EV(`'000)


D1 X 190 0.6 114
D2 Y 200 0.2 40
D3 Y 160 0.2 32
EV with Perfect Information 186

So, EV of the Perfect Information = 186 - 140 = ` 46 i.e. ` 46,000

7. Pioneer Projects Ltd. is considering accepting one of two mutually exclusive projects X & Y. The cash flow
and probabilities are estimated as under:
Project X Project Y
Probability Cash flow Probability Cash flow
0.10 12,000 0.10 8,000
0.20 14,000 0.25 12,000
0.40 16,000 0.30 16,000
0.20 18,000 0.25 20,000
0.10 20,000 0.10 24,000
Advise Pioneer Projects Ltd.
Directorate of Studies, The Institute of Cost Accountants of India (Statutory Body under an Act of Parliament) Page 22
Work Book: Strategic Financial Management

Solution:

Calculation of Standard Deviation Project X


P X EV = P*X (x-x) ('000) (x - x)2 P(x - x)2
0.10 12,000 1,200 -4 16 1.6
0.20 14,000 2,800 -2 4 0.8
0.40 16,000 6,400 0 0 0
0.20 18,000 3,600 2 4 0.8
0.10 20,000 2,000 4 16 1.6
x = 16,000 Variance = 4.8

Standard Deviation () = 4.8 = 2.19


Coefficient of Variation = /EV X 100 = 2.19/16 X100= 13.68%

Calculation of Standard Deviation Project Y


P X EV = P*X (x-x) ('000) (x - x)2 P(x - x)2
0.10 8,000 800 -8 64 6.4
0.25 12,000 3,000 -4 16 4.0
0.30 16,000 4,800 0 0 0
0.25 20,000 5,000 4 16 4.0
0.10 24,000 2,400 8 64 6.4
x = 16,000 Variance = 20.8

Standard Deviation () = 20.8 = 4.56


Coefficient of Variation = /EV X 100 = 4.56/16 X100= 28.58%

Analysis - Project Y is more risky as it is more susceptible to wider degree of variation around the most likely
outcome than Project X. Therefore, Project X should be preferred.

8. A company is trying to choose between two investment proposals A and B. Project A has a standard
deviation of ` 6,500 while Project B has a standard deviation of ` 7,200. The finance manager wishes to know
which investment to choose, given each of the following combinations of the expected values;
(i) Project A and Project B both have expected net present value of ` 15,000.
(ii) Project A has expected NPV of ` 18,000 while for Project B it is ` 22,000.

Solution:

(i) If Project A and Project B both have expected net present value of ` 15,000, the Finance Manager
should select Project A since its Standard Deviation is lesser than that of Project B. The lesser Standard
Deviation represents lesser risk.

Directorate of Studies, The Institute of Cost Accountants of India (Statutory Body under an Act of Parliament) Page 23
Work Book: Strategic Financial Management

(ii) If Project A has expected NPV of ` 18,000 while for Project B is ` 22,000, then selection of Project will be
done with the help of Coefficient of Variation.
Coefficient of Variation = Standard Deviation/ Expected NPV
Project A = 6500/18000 = 0.361
Project B = 7200/22000 = 0.327
Analysis - Investment in Project B should be chosen, since its Coefficient of Variation is lower.

9. M Ltd. is attempting to decide whether or not to invest in a project that requires an initial outlay of ` 4 lakhs.
The cash flows of the project are known to be made up of two parts, one of which varies independently over
time and the other one which display perfect positive correlation. The cash flows of the six year life of the
project are:
(`)
Perfectly Correlated Components Independent Component
Year Mean Standard Deviation Mean Standard Deviation
1 40,000 4,400 42000 4000
2 50,000 4,500 50000 4400
3 48,000 3,000 50000 4800
4 48,000 3,200 50000 4000
5 55,000 4,000 52000 4000
6 60,000 4,000 52000 3600

(i) Find out the expected value of the NPV and its standard deviation, using a discount rate of 10%
(ii) Also find the probability that the project will be successful, i.e. P (NPV > 0) and state the assumptions
under which this probability can be determined.

Solution:

(a) Calculation of NPV


Year Mean (Perfectly correlated Mean (Independent Expected PV factor @ Present value
component) component) Value 10%
(1) (2) (3) (4) = (2) + (3) (5) (6)=(4)X(5)
1 40,000 42,000 82,000 0.909 74,538
2 50,000 50,000 1,00,000 0.826 82,600
3 48,000 50,000 98,000 0.751 73,598
4 48,000 50,000 98,000 0.683 66,934
5 55,000 52,000 1,07,000 0.621 66,447
6 60,000 52,000 1,12,000 0.564 63,168
4,27,285
Less: Cash outflow 4,00,000
Expected Net Present Value 27,285

Directorate of Studies, The Institute of Cost Accountants of India (Statutory Body under an Act of Parliament) Page 24
Work Book: Strategic Financial Management
Calculation of Standard Deviation for Perfectly Correlated Components
Year Standard Deviation PV factor @ 10% Present Value
1 4,400 0.909 3,999.6
2 4,500 0.826 3,717.0
3 3,000 0.751 2,253.0
4 3,200 0.683 2185.6
5 4,000 0.621 2484.0
6 4,000 0.564 2256.0
16895.2

Standard Deviation = Variance = (16.895.2)2 = 285447783

Calculation of Variance for Independent Component


Year (1) Standard Deviation (2) PV Factor @ 10% (3) Present Value (4)=(2)X(3) (Present Value)2 (5)
1 4,000 0.909 3,636.0 1,32,20,496
2 4,400 0.826 3,634.4 1,32,08,863
3 4,800 0.751 3,604.8 1,29,94,583
4 4,000 0.683 2,732.0 74,63,824
5 4,000 0.621 2,484.0 61,70,256
6 3,600 0.564 2,030.4 41,22,524
Variance = 5,71,80,546

Variance of the Project


= Variance of Perfectly Correlated Components + Variance of Independent Components = (16,895.2) 2 + `
5,71,80,546 = 28,54,47,783 + 5,71,80,546 = ` 34,26,28,329
Standard Deviation () = 342628329 = 18,510

0−27285
(ii) P (NPV≥ 0) = P (z ≥ ) = P (z ≥ -1.47) = 0.5+0.4292(from normal table) = 0.9292
18510

Hence, the probability that the project will be successful is 92.92%. The assumption made under which this
probability can be determined is that the cash flows follow normal distribution with mean (M) is 27,285 and
standard deviation (a) is 18,510 as calculated above.

10. From the following project details calculate the sensitivity of the (a) Project cost, (b) Annual cash flow, and
(c) Cost of capital. Which variable is the most sensitive?

Project cost ` 12,000 Annual cash flow ` 4,500


Life of the project 4 years Cost of capital 14%

The annuity factor at 14% for 4 years is 2.9137 and at 18% for 4 years is 2.6667.

Directorate of Studies, The Institute of Cost Accountants of India (Statutory Body under an Act of Parliament) Page 25
Work Book: Strategic Financial Management
Solution:

Particulars `
Annual cash inflow (4500 X 2.9137) 13,112
Less: Project cost 12,000
Net present value 1,112

(i) Sensitivity for Project Cost


If the project cost is increased by ` 1112, the NPV of the project will become zero. Therefore, the sensitivity
for project cost is = 1,112/12000 X 100 = 9.27%

(ii) Sensitivity for Annual Cash Inflow


If the present value of annual cash inflow is lower by ` 1112, the NPV of the project will become zero.
Therefore, the sensitivity for annual cash flow is = 1112/13112 X 100 = 8.48%

(iii) Sensitivity for Cost of Capital


Let ‗x' be the annuity factor which gives a zero NPV i.e. 'x' is the IRR
-12,000 + 4,500x = 0
Or, 4,500x = 12,000
Or, x = 12,000/4,500 = 2.6667

Hence, x = 2.6667 and at 18% for 4 years, the annuity factor is 2.6667.
Sensitivity % = (18% -14%)/14% = 29%

Analysis: The cash inflow is more sensitive, since only 8.5% change in cash inflow will make the NPV of the
project zero.

11. Determine the risk adjusted net present value of the following projects:
Particulars A B C
Net cash outlay (`) 1,00,000 1,20,000 2,10,000
Project life 5 years 5 years 5 years
Annual cash inflow (`) 30,000 42,000 70,000
Coefficient of variation 0.4 0.8 1.2

The company selects the risk-adjusted rate of discount on the basis of the co-efficient of variation:
Coefficient of variation Risk adjusted rate of Present value factor 1 to 5 years at risk adjusted
discount rate of discount
0.0 10% 3.791
0.4 12% 3.605
0.8 14% 3.433
1.2 16% 3.274
1.6 18% 3.127
2.0 22% 2.864
More than 2.0 25% 2.689

Directorate of Studies, The Institute of Cost Accountants of India (Statutory Body under an Act of Parliament) Page 26
Work Book: Strategic Financial Management

Solution:

Statement Showing Computation of Risk Adjusted Net Present Value


Project A B C
Net cash outlay (i) 100000 120000 210000
Annual cash inflow (ii) 30000 42000 70000
Present value factor 1 to 5 years at risk adjusted rate of discount (iii) 3.605 3.433 3.274
Present value of cash inflow (iv) = (ii)*(iii) 108150 144186 229180
Risk adjusted NPV (v) = (iv) – (i) 8150 24186 19180

Directorate of Studies, The Institute of Cost Accountants of India (Statutory Body under an Act of Parliament) Page 27
Work Book: Strategic Financial Management

Study Note – 3

Leasing Decisions

1. A factory needs equipment for use. It has the option of outright purchase or leasing the equipment. Data are
given below. Recommend the best option that the factory should choose.

Option 1
Purchase outright for a cost of ` 80 lakhs. It is to be entirely financed by a term loan @18% p.a. interest on
outstanding payable on a yearly basis. The term loan to be repaid in eight equal instalments of ` 10 lakhs
each, beginning from second year-end. The economic life of the equipment is assessed to be ten year. The
equipment will be depreciated @ 10% p.a. on straight line basis, with insignificant salvage value at the end
of the economic life? The estimated maintenance expenses would be as detailed below:
Year 1 2 3 4 5 6 7 8 9 10
MC* 4.00 4.40 4.88 5.47 6.18 7.05 8.11 9.41 11.01 13.00
(*) MC- Maintenance cost in ` lakhs.

Option 2
The equipment may be leased for a ten-year period. The maintenance of the equipment will be done by the
lessor. The lessee has to pay ` 18 lakhs annual rental at the beginning of each year over the lease period.
Note - Assume that the lessee is in a tax bracket of 50% and average cost of capital of the lessee firm as
14% p.a.

Solution:

Option I: Purchase (` lakhs)


Year Loan Amount Interest on Mainte- Interest + Tax Outflow Total
repaid balance balance nance Maintenance + saved Interest + outflow
Cost Depreciation 50% Maintenance
1 - 80 14.40 4.0 26.40 13.20 5.20 5.20
2 10 70 14.40 4.40 26.80 13.40 5.40 15.40
3 10 60 12.60 4,88 25.48 12.74 4.74 14.74
4 10 50 10.80 5.47 24.27 12.13 4.14 14.14
5 10 40 9.00 6.18 23.18 11.59 3.59 13.59
6 10 30 7.20 7.05 22.25 22.25 11.13 13.13
7 10 20 5.40 8.11 21.51 10.76 2.76 12.76
8 10 10 3.60 9.41 21.01 10.50 2.50 12.50
9 10 0 1.80 11.01 20.81 10.41 2.41 12.41
10 - - - 13.00 21.00 10.50 2.50 2.50

Directorate of Studies, The Institute of Cost Accountants of India (Statutory Body under an Act of Parliament) Page 28
Work Book: Strategic Financial Management

Calculation of Present Value (` lakhs)


Year Total cash outflow DCF @ 14% Present value
1 5.20 0.877 4.56
2 15.40 0.769 11.84
3 14.74 0.675 9.95
4 14.14 0.592 8.37
5 13.59 0.519 7.05
6 13.13 0.465 6.11
7 12.76 0.400 5.10
8 12.50 0.351 4.39
9 12.41 0.308 3.82
10 2.50 0.270 0.67
Total present value of cash outflows 61.86

Option II : Lease (` lakhs)


Year Lease rent Lease rent after tax shield DCF @ 14% Present value
1 18 9 1.000 9.00
2 18 9 0.877 7.89
3 18 9 0.769 6.92
4 18 9 0.675 6.07
5 18 9 0.592 5.33
6 18 9 0.519 4.67
7 18 9 0.465 4.19
8 18 9 0.400 3.60
9 18 9 0.351 3.16
10 18 9 0.308 2.77
Total present value of cash outflows 53.60

Suggestion: The present value of net cash flows is lowest for lease option, hence it is suggested to take
equipment on lease basis.

2. A firm has the choice of buying a piece of equipment at a cost of ` 1,00,000 with borrowed funds at a cost of
18% p.a. repayable in five annual instalments of ` 32,000, or to take on lease the same on an annual rental
of ` 32,000. The firm is in the tax-bracket of 40%.
Assume:
(i) The salvage value of the equipment at the end of the period is zero.
(ii) The firm uses straight line depreciation.

Discounting factors are:


@ 9% 0.917 0.842 0.772 0.708 0.650
@ 11% 0.901 0.812 0.731 0.659 0.593
@ 18% 0.847 0.718 0.609 0.516 0.437
Which alternative do you recommend?

Directorate of Studies, The Institute of Cost Accountants of India (Statutory Body under an Act of Parliament) Page 29
Work Book: Strategic Financial Management

Solution:

Cost of Borrowed Funds:


I (l-T) = 1896(1 -0.40)
= 1896(0.60)
= 10.8% say 11%

Computation of Cost of Owning (`)


Years Annual payment Interest Amortization Depreciation Tax savings Cost of owning
1 32,000 18,000 14,000 20,000 15,200 16,800
2 32,000 15,480 16,520 20,000 14,192 17,808
3 32,000 12,506 19,494 20,000 13,002 18,998
4 32,000 8,997 23,003 20,000 11,599 20,401
5 31,840 4,857 26,983 20,000 9,943 21,897
Total 1,59,840 59,840 1,00,000 1,00,000 63,936 95,904

Incremental cost of leasing over cost of owning (`)


Years Cost of Owning Net lease Case Advantage of owning D.F. @ 11% Present value of advantage
1 16,800 19,200 2,400 0.901 2,162
2 17,808 19,200 1,392 0.812 1,130
3 18,998 19,200 202 0.731 148
4 20,401 19,200 (1,201) .0.659 (791)
5 21,897 19,200 (2,697) 0.593 (1,599)
Total 95,904 96,000 96 1,050

Suggestion: It is advantageous to purchase the asset on borrowed funds, as the present value of
advantages is positive.

3. PQR. Ltd. is considering the possibility of purchasing a multipurpose machine which cost ` 10 lakhs. The
machine has an expected life of 5 years. The machine generates ` 6 lakhs per year before depreciation
and tax, and the management wishes to dispose the machine at the end of 5 years which will fetch ` 1 lakh.
The depreciation allowable for the machine is 25% on written down value and the company's tax rate is
50%. The company approached a NBFC for a five year lease for financing the asset which quoted a rate of `
28 per thousand per month. The company wants you to evaluate the proposal with purchase option. The
cost of capital of the company is 12% and for lease option it wants you to consider a discount rate of 16%.

Directorate of Studies, The Institute of Cost Accountants of India (Statutory Body under an Act of Parliament) Page 30
Work Book: Strategic Financial Management
Solution:

Evaluation of Purchase Option (` lakhs)


Particulars 0 1 2 3 4 5
Initial outlay (10) - - - - -
Operating Profit 6.00 6.00 6.00 6.00 6.00
Less : Depreciation 2.50 1.88 1.40 1.06 0.79
Profit before tax 3.50 4.12 4.60 4.94 5.21
Less : Tax (a.1 50% 1.75 2.06 2.30 2.47 2.60
Profit after tax 1.75 2.06 2.30 2.47 2.61
Add : Depreciation 2.50 1.88 1.40 1.06 0.79
Salvage value of machine - - 1.00
Net cash Inflow 4.25 3.94 3.70 3.53 4.40
Present value factor @ 12% 1.00 0.893 0.797 0.712 0.636 0.567
Present values (10) 3.80 3.14 2.63 2.25 2.49
Net present value of the purchase option is ` 431000

Evaluation of Lease Option (` lakhs)


Particulars 1 2 3 4 5
Operating profit 6.00 6.00 6.00 6.00 6.00
Less ; Lease rent 3.36 3.36 3.36 3.36 3.36
Profit before tax 2.64 2.64 2.64 2.64 2.64
Tax @ 50% 1.32 1.32 1.32 1.32 1.32
Profit after tax 1.32 1.32 1.32 1.32 1.32
Discount factor @ 16% 0.862 0.743 0.641 0.552 0.476
Present values 1.14 0.98 0.85 0.73 0.63
The net present value of lease option is ` 4,33,000.

Suggestion: From the analysis of the above we can observe that NPV of lease option is more than that of
purchase option. Hence, lease of machine is recommended.

4. XYZ Ltd. is considering a proposal to acquire an equipment costing ` 5,00,000. The expected effective life of
the equipment is 5 years. The company has two options - either to acquire it by obtaining a loan of ` 5 lakhs
at 12% interest p.a. or by lease. The following additional information is available:
(i) the principal amount of loan will be repaid in 5 equal yearly instalments.
(ii) the full cost of the equipment will be written off over a period of 5 years on straight line basis and it is to
be assumed that such depreciation charge will be allowed for tax purpose.
(iii) the effective tax rate for the company is 40% and the after tax cost of capital is 10%.
(iv) the interest charge, repayment of principal and the lease rentals are to be paid on the last day of each
year.
You are required to work out the amount of lease rental to be paid annually, which will match the loan
option.

Directorate of Studies, The Institute of Cost Accountants of India (Statutory Body under an Act of Parliament) Page 31
Work Book: Strategic Financial Management
Solution:

Calculation of Interest under Loan Option and Depreciation (`)


Year Principal amount at Repayment Principal at Interest for year Depreciation. for
beginning of year at end of year end of year @ 12% year
1 5,00,000 1,00,000 4,00,000 60,000 1,00,000
2 4,00,000 1,00,000 3,00,000 48,000 1,00,000
3 3,00,000 1,00,000 2,00,000 36,000 1,00,000
4 2,00,000 1,00,000 1,00,000 24,000 1,00,000
5 1,00,000 1,00,000 Nil 12,000 1,00,000

Calculation of Present Value under Loan Option (`)


Year Repayment Interest Total Tax on Interest Total Net outflow Discount NPV
of principal on loan (1)+(2) depreciation (a)+(b) (3)-(c) factor
(1) (2) (3) (a) (b) (c)
1 1,00,000 60,000 1,60,000 40,000 24,000 64.000 96,000 0.909 87,264
2 1,00,000 48,000 1,48,000 40,000 19,200 59,200 88,800 0.826 73,349
3 1,00,000 36,000 1,36,000 40,000 14,400 54,400 81,600 0.751 61,282
4 1,00,000 24,000 1,24,000 40,000 9,600 49,600 74,400 0.683 50,815
5 1,00,000 12,000 1,12,000 40,000 4,800 44,800 67,200 0.621 41,731
Total present value of cash outflows 3.790 3,14,441

Annual cash outflow after-tax = 3,14,441/3.790 = ` 82,966


Annual lease rental which will be indifferent to loan option = 82,966/1 - 0.40 = ` 1,38,277

5. N Ltd. is a hire purchase and leasing company. It has been approached by a small business firm interested
in acquiring a machine through leasing. The quoted price of the machine is ` 5,00,000. 10% sale tax is extra.
The lease will be for a primary lease period of 5 years.

The finance company wants 8% post-tax return on the outlay. Its effective tax rate is 35%. The income tax
rate of depreciation on the machine is 25% (WDV). Lease rents are payable in arrear at the end of each
year.

Calculate the annual rent to be charged by N Ltd.

Solution:

Determination of Cash outflows (`)

Cost of machine inclusive of sale tax (10%) 5,50,000


Less: Tax saving on Depreciation (Tax shield Relief) 1,22,284
Present value of cash outflows for purchase 4,27,716

Directorate of Studies, The Institute of Cost Accountants of India (Statutory Body under an Act of Parliament) Page 32
Work Book: Strategic Financial Management
Computation of Tax Saving on Depreciation of the Machine
Year Cost/WDV Depreciation @ 25% Tax @ 35% PV factor @ 8% P.V. of Dep. tax shield
1 5,50,000 1,37,500 48,125 0.926 44,564
2 4,12,500 1,03,125 36,094 0.857 30,933
3 3,09,375 77,344 27,070 0.794 21,494
4 2,32,031 58,008 20,303 0.735 14,923
5 1,74,023 43,506 15,227 0.681 10,370
3.993 1,22,284

Calculation of Leasing Rent


Let, the required lease rent per year be 'x'
Post-tax rental income p.a. (1 - 0.35) x = 0.65x
P.V. of 5 year's post-tax rental income = 0.65x  3.993 = 2.59545x

This sum should be equal to ` 4,27,716


2.59545x = 4,27,716
x = 4,27,716/2.59545 = 1,64,795

Hence, the annual rent to be charged by N Ltd. is ` 1,64,795.

6. S Ltd. is faced with a decision to purchase or acquire on lease a mini car. The cost of the mini car is `
1,26,965. It has a life of 5 years. The mini car can be obtained on lease by paying equal lease rentals
annually. The leasing company desires a return of 10% on the gross value of the asset. S Limited can also
obtain 100% finance from its regular banking channel. The rate of interest will be 15% p.a. and the loan will
be paid in five annual equal instalments, inclusive of interest. The effective tax rate of the company is 40%.
For the purpose of taxation it is to be assumed that the asset will be written off over a period of 5 years on a
straight line basis.
(a) Advise S Ltd.about the method of acquiring the car.
(b) What should be the annual lease rental to be charged by the leasing company to match the loan
option?

For your exercise use the following discount factors:


Discount Rate Year 1 Year 2 Year3 Year 4 Year 5
10% 0.91 0.83 0.75 0.68 0.62
15% 0.87 0.76 0.66 0.57 0.49
9% 0.92 0.84 0.77 0.71 0.65

Solution:

(a) Annual loan repayment = Loan amount/ Annuity factor of 15% = 126965/3.86 = ` 32892

Note - Annuity factor is based on the assumption that loan instalment is repaid at the beginning of the year
to be at par with lease rentals. Such annuity factor at 15% works out to be 3.86.

Directorate of Studies, The Institute of Cost Accountants of India (Statutory Body under an Act of Parliament) Page 33
Work Book: Strategic Financial Management
Computation of Interest in Debt Payments (`)
Year 0 1 2 3 4
Opening balance of principal Interest @ 15% 1,26,965 94,073 75,292 53,694 28,856
Total Repayment of instalment Closing balance Nil 14,111 11,294 8,054 4,036*
1,26,965 1,08,184 86,586 61,748 32,892
32,892 32,892 32,892 32,892 32,892
94,073 75,292 53,694 28,856 Nil

*Difference between the instalment amount and opening balance of 4th year.

Schedule of Cash Outflows in Debt Financing (`)


End Loan Interest Depreciation Tax. shield Net cash PV factor P.V. of cash
of re-payment @15% [(2) + (3) X 0.40] Outflows - (4) @ 9% outflows
(1) (2) (3) (4) (5) (6) (7)
0 32,892 - - - 32,892 1.00 32.892
1 32,892 14,111 25,393 15,802 17,090 0.92 15,723
2 32,892 11,294 25,393 14,675 18,217 0.84 15,302
3 32,892 8,054 25.393 13,379 19,513 0.77 15,025
4 32,892 4,036 25,393 11,772 21,120 0.71 14,995
5 - 25,393 10,157 (10,157) 0.65 (6,602)
Total present value of cash outflows 87,335
Annual lease rental = Cost of the asset/Annuity factor of 10%= 126965/4.17 = `30447

Schedule of cash outflows - Leasing Alternative (`)


End of the year Lease payment Tax shield After tax cash PV factors Present value
outflows at 9% of cash outflows
0 30,447 - 30,447 1.00 30,447
1-4 30,447 12,179 18,268 3.24 59,188
5 12,179 (12,179) 0.65 (7,916)
Total present value of cash outflows ` 81,719

Decision - The present value of cash outflows under lease financing is ` 81,719 while that of debt financing
(i.e., owning the asset) is ` 87,335. Thus leasing has an advantage over ownership in this case.

(b) Let the Annual Lease Rentals be 'x'

Therefore the after tax cost of lease rentals will be 0.60 x


Present value will be 0.60 x  4.17 = 2.502 x
Equating 2.502 x = ` 87,335
The value of x is obtained at ` 34,906.

Therefore, the lease rental should be ` 34,906 to match the loan option.

Directorate of Studies, The Institute of Cost Accountants of India (Statutory Body under an Act of Parliament) Page 34
Work Book: Strategic Financial Management

Study Note – 4

Institutions in Finance Markets

1. Briefly discuss the important functions of a Financial System

Answer:

The following are the functions of a Financial System:


(i) Mobilise and allocate savings: The financial system links the savers and investors to mobilise and
allocate the savings efficiently and effectively.
(ii) Monitor corporate performance: The operators of the financial system not only select the projects that
are to be funded but also monitor the performance of the investment carefully.
(iii) Provide payment and settlement systems: The financial system provides adequate payment and
settlement system to its investors for exchange of goods and services and transfer of economic
resources through time and across geographic regions and industries. The depositories and clearing
houses are in charge of the clearing and settlement mechanism of the stock markets.
(iv) Optimum allocation of risk-bearing and reduction: The financial system provides various option of risk-
reduction to its investors such as diversified portfolios and also by framing rules governing the operation
of the system.
(v) Disseminate price-related information: The financial system helps in disseminating the price related
information so that the investors can take well informed decisions regarding the investment,
disinvestment, reinvestment or holding of any particular asset.
(vi) Offer portfolio adjustment facility – The financial system also provides portfolio adjustment facility by
providing the options of buying and selling a wide variety of financial assets in a quick, cheap and
reliable way.
(vii) Lower the cost of transactions: The transactions done within the financial system are smooth, effective
and have lower costs.
(viii) Promote the process of financial deepening and broadening –Financial deepening refers to an
increase of financial assets as a percentage of GDP. Financial depth is an important measure of
financial system development as it measures the size of the financial intermediary sector. Financial
broadening refers to building an increasing number of varieties of participants and instruments. The
financial system thus promotes the process of financial deepening and broadening.

2. What are the tools and techniques used by RBI to maintain financial stability?

Answer:

The following tools and techniques used by RBI to maintain financial stability:
(i) Financial Stress Indicator: It is a contemporaneous indicator of conditions in financial markets and in
the banking sector.
Directorate of Studies, The Institute of Cost Accountants of India (Statutory Body under an Act of Parliament) Page 35
Work Book: Strategic Financial Management
(ii) Systemic Liquidity Indicator: This is used for assessing stresses in availability of systemic liquidity.
(iii) Fiscal Stress Indicator: This indicator used for assessing build up of risks from the fiscal.
(iv) Network Model: This model used for the bilateral exposures in the financial system - for assessing the
inter- connectedness in the system.
(v) Banking Stability Indicator: This indicator is used for assessing risk factors having a bearing on the
stability of the banking sector.
(vi) A series of Banking Stability Measures for assessing the systemic importance of individual banks.

3. What are the direct instruments used by the RBI to formulate and implement monetary policy?

Answer:

There are several direct instruments that are used in the formulation and implementation of monetary
policy. These are as follows:
(i) Cash Reserve Ratio (CRR): The share of net demand and time liabilities that banks must maintain as cash
balance with the Reserve Bank. The Reserve Bank requires banks to maintain a certain amount of cash
in reserve as percentage of their deposits to ensure that banks have sufficient cash to cover customer
withdrawals. The adjustment of this ratio, is done as an instrument of monetary policy, depending on
prevailing conditions. Our centralized and computerized system allows for efficient and accurate
monitoring of the balances maintained by banks with the Reserve Bank of India.
(ii) Statutory Liquidity Ratio (SLR): The share of net demand and time liabilities that banks must maintain in
safe and liquid assets, such as government securities, cash and gold.
(iii) Refinance facilities: Sector-specific refinance facilities (e.g., against lending to export sector) provided
to banks exchange or other commercial papers. It also signals the medium-term stance of monetary
policy.

4. Mention some of the important regulations relating to acceptance of deposits by NBFCs.

Answer:

Some of the important regulations relating to acceptance of deposits by NBFCs are as under:
(i) The NBFCs are allowed to accept/renew public deposits for a minimum period of 12 months and
maximum period of 60 months. They cannot accept deposits repayable on demand.
(ii) NBFCs cannot offer interest rates higher than the ceiling rate prescribed by RBI from time to time. The
present ceiling is 12.5 per cent per annum. The interest may be paid or compounded at rests not
shorter than monthly rests.
(iii) NBFCs cannot offer gifts/incentives or any other additional benefit to the depositors.
(iv) NBFCs (except certain AFCs) should have minimum investment grade credit rating.
(v) The deposits with NBFCs are not insured.
(vi) The repayment of deposits by NBFCs is not guaranteed by RBI.
(vii) Certain mandatory disclosures are to be made about the company in the Application Form issued by
the company soliciting deposits.

Directorate of Studies, The Institute of Cost Accountants of India (Statutory Body under an Act of Parliament) Page 36
Work Book: Strategic Financial Management

Study Note – 5

Instruments in Financial Markets

1. Choose the correct alternative:

(i) A person can earn 12 per cent by investing in equity shares on his own. Now he is considering a recently
announced equity based mutual fund scheme in which initial expenses 5 per cent and annual recurring
expenses are 1.5 per cent. How much should the mutual fund earn to provide him a return of 10 per
cent?
a) 11.04 %
b) 12.02%
c) 12.63%
d) 12.98%

(ii) XYZ mutual fund had a net asset value of ₹10 at the beginning of a month, made income and capital
gain distribution of ₹0.05 and ₹0.02 respectively per unit during the month. The fund ended the month
with a net asset value of ₹10.08. The monthly rate of return of XYZ mutual fund is-
a) 1.5%
b) 1.9%
c) 2.0%
d) 2.2%

(iii) The following information is extracted from ABC Mutual Fund Scheme:
Asset Value at the beginning of the month ₹60
Annualized return 12 %
Distributions made in the nature of Income ₹0.40 and ₹ 0.30
and Capital gain (per unit respectively).
The month end net asset value of the mutual fund scheme is (limit your answers to two decimals)-
a) ₹ 58.38
b) ₹ 59.05
c) ₹59.90
d) ₹ 60.30

(iv) The following information is available to a mutual fund scheme:


Size of the scheme ₹150 Lakhs
Face value of the shares ₹100
Number of the outstanding shares ₹1.5 Lakhs
Market value of the fund’s investments ₹220 Lakhs
Receivables ₹1 Lakhs
Liabilities ₹50,000
Directorate of Studies, The Institute of Cost Accountants of India (Statutory Body under an Act of Parliament) Page 37
Work Book: Strategic Financial Management
The NAV of the scheme is-
a) ₹ 145.00
b) ₹145.90
c) ₹146.60
d) ₹147.00

(v) In case of an open ended Mutual Fund scheme the market price (ex-dividend) was ₹ 70. A dividend of
₹10 has just been paid and ex-divided price now is ₹ 82. The return has earned over the past year by the
mutual fund is-
a) 30.30%
b) 30.90%
c) 31.00%
d) 31.43%

(vi) Money Plant mutual fund had a Net Asset Value (NAV) of ₹ 60 at the beginning of the year. During the
year a sum of ₹ 6 was distributed as dividend besides ₹ 2 as capital gains distribution. At the end of the
year NAV was ₹72. The total return for the year-
a) 33.33 %
b) 33.95%
c) 34.23%
d) 34.78%

(vii)Suppose the aforesaid mutual fund [question (iv)] in the next year gives a dividend of ₹4 and no capital
gains distribution and NAV at the end of second year is ₹66. So, the return for the second year would be–
a) 15.96 %
b) 16.66 %
c) 16.98 %
d) 17.16 %

(viii) Following information is available regarding a mutual fund:


Return 13%
Risk (S.D. i.e. σ) 15%
Beta (ß) 0.90
Risk Free Rate 10%

The Sharpe Ratio of the mutual fund is


(a) 0.20
(b) 0.25
(c) 0.30
(d) 0.35

Directorate of Studies, The Institute of Cost Accountants of India (Statutory Body under an Act of Parliament) Page 38
Work Book: Strategic Financial Management
Solution:

Question No. i ii iii iv v vi vi viii


Answer b a c d d a b a

2. Find out Net Asset Value (NAV) per unit from the following information of Scheme Grow Money.

Name of the scheme Grow Money


Size of the scheme ₹ 250 Lakh
Face value of the shares ₹ 10
Number of the outstanding shares 2.5 Lakh
Market value of the fund’s investments ₹ 160 Lakh
Cash and other assets in hand ₹ 1 Lakh
Receivables ₹ 3 Lakh
Liabilities ₹1.2 Lakh

Solution:

Net Asset Value= (Total Assets – Liabilities) / No. of shares


Total Assets =
Market value of the fund‘s investments ₹160 Lakh
Cash and other assets in hand ₹ 1 Lakh
Receivables ₹ 3 Lakh
Total ₹ 164

Liabilities
Liabilities ₹ 1.2 Lakh

NAV= (₹164Lakh-1.2Lakh )/2.5Lakh Shares =₹ 65.12

3. The following portfolio details of a mutual fund scheme are given below:
Stock No. of shares Price (₹)
P 4 Lakh 45
Q 6 Lakh 50
R 8 Lakh 25
S 12 Lakh 30

The scheme has accrued expenses towards portfolio managers of ₹ 6 Lakh. There are 80 lakh shares
outstanding. Find out the NAV (Net Asset Value) per unit of the scheme.

Directorate of Studies, The Institute of Cost Accountants of India (Statutory Body under an Act of Parliament) Page 39
Work Book: Strategic Financial Management
Solution:

Portfolio of the Scheme


Stock No. of shares Price (₹) Value (₹)
P 4 Lakh 45 180 Lakh
Q 6 Lakh 50 300 Lakh
R 8 Lakh 25 200 Lakh
S 12 Lakh 30 360 Lakh
Total 1040 Lakh

NAV per Unit= (Total Portfolio – Total Expenses)/No of Shares Outstanding


= (₹1040 Lakh-₹6 Lakh)/ 80 Lakh = ₹12.925

4. Vibrant Mutual Fund company made an issue of 10,00,000 units of ₹10 each on 01.01.2017. No entry load
was charged. It made the following investments:
Particulars
50,000 Equity Shares of ₹100 each @ ₹150 ₹ 75,00,000
7% Government Securities ₹ 10,00,000
8% Debentures (Unlisted) ₹5,00,000
10% Debentures (Listed) ₹ 6,00,000
Total ₹ 96,00,000

During the year, dividends of ₹10,00,000 were received on equity shares. Besides, interest on all types of
debt securities was received on due time. At the end of the year equity shares and 10% debentures are
quoted at 200% and 90% respectively. Other investments are quoted at par.

Find out the Net Asset Value (NAV) per unit given that the operating expenses during the year amounted to
₹ 6,00,000. Also find out the NAV, if the Mutual Fund had distributed a dividend of ₹1 per unit during the year
to the unit holders.

Solution:

Given the Total initial investments is ` 96,00,000, out of issue proceeds of `1,00,00,000. Therefore, the balance of
`4,00,000 is considered as issue expenses.
Particulars Opening Value of Capital Closing Value of Income
Investments (₹) Appreciation (₹) Investments (₹) (₹)
Equity Shares 75,00,000 25,00,000 1,00,00,000 10,00,000
7% Government Securities 10,00,000 Nil 10,00,000 70,000
8% Debentures (Unlisted) 5,00,000 Nil 5,00,000 40,000
10% Debentures (Listed) 6,00,000 -60,000 5,40,000 60,000
Total 96,00,000 24,40,000 1,20,40,000 11,70,000

Directorate of Studies, The Institute of Cost Accountants of India (Statutory Body under an Act of Parliament) Page 40
Work Book: Strategic Financial Management
Less: Operating expenses during the period (6,00,000)
Net income 5,70,000
Net fund balance = ₹ ( 1,20,40,000 + 5,70,000) 1,26,10,000
Less : Dividend `10,00,000 (10,00,000 unit @ ₹1 per unit) 10,00,000
Net fund balance after (dividend) 1,16,10,000
NAV (before calculating dividend) = 1,26,10,000/10,00,000 12.61
NAV (after calculating dividend) = 1,16,10,000/10,00,000 11.61

5. ABC Company Limited as the Asset Management Company (Commencing its functions from 1 st May, 2017)
under a trust deed with T.P Mutual Funds Limited managing solely equity schemes of 8 years (with effect
from 1st June, 2017). Information relating to its expenses incurred during the year 2018-19 is as follows.
₹ in Crores
i. Custodian Charges 0.59
ii. Brokerage and Transaction Cost 15.31
iii. Agents Commission 32.95
iv. Audit Fees 1.82
v. Initial Issue Expenses 112.5

If accounts are intended to be closed on 31st March, 2019 and on that date the AMC is expected to hold Net
Assets worth ₹1,500 crores (For operational year 2018-19).
What would be the eligible amount of expense chargeable by the AMC for its operations?

Solution:

From the given problem it is clear that ABC Company Limited as AMC is engaged in issuance of a close ended
scheme (As the maturity period of the fund lies between 3 – 15 years). Expenses related to issue of units are to
be amortized over the period of the scheme on weekly basis.

The total number of weeks within which the scheme remains effective is of (8 years × 12 months × 4 weeks) = 384
weeks. The financial year (i.e. 2018-19) comprise of (1 year × 12 months × 4 weeks) = 48 weeks.
Issue Expenses eligible for amortization in the year 2018-19 is of ₹ (48 × 112.5) / 384 = ₹14.0625 crores.

Maximum Limit of charging expenses for management by any AMC is 1.5% of the average weekly net assets
held up to the ceiling of ₹ 100 crores (excluding amortized part of expenses on issue and redemption). For
financial year 2018-19 assets held by the AMC is of ₹ 1,500 crores, therefore average weekly net assets held = ₹
1,500 / 48 = ₹ 31.25 crores.

Maximum permissible expenses for management by ABC Company Ltd. is of ₹ (31.25 × 1.5%) = ₹ 0.46875 crores.
Total threshold or simply total of maximum eligible expense for management = ₹ (14.0625 + 0.46875) crores =
₹14.53125 crores.

Directorate of Studies, The Institute of Cost Accountants of India (Statutory Body under an Act of Parliament) Page 41
Work Book: Strategic Financial Management
Table showing actual expenses claimed from AMC‗s part from T.P Mutual Funds Limited
(₹ in Crores)
i. Custodian Charges 0.59
ii. Brokerage and Transaction Cost 15.31
iii. Agents Commission 32.95
iv. Audit Fees 1.82
v. Initial Issue Expenses 14.0625
vi. Total Expenses of Management 64.7325

Therefore eligible amount of expenses to be charged against management of scheme by ABC Company
Limited is ₹ 14.53125 crores.

6. Goodluck Mutual Funds Limited registered in Mumbai issues equity oriented Mutual Fund Schemes mostly
traded in BSE as well as in CSE. The trade being in operations found to cease on 8 th February, 2018 in the
respective exchanges.

Prices of the units as on the date are as follows:


Stock Exchange Opening Price (₹) High (₹) Low (₹) Closing Price (₹)
BSE 110.25 111.25 110.45 110.8
CSE 110.5 110.95 110.25 110.75

Additional Information:
 Uncertain Mutual Funds Limited having a more or less similar portfolio of risk-return manages to offer a
dividend yield of 5.46% at a payout ratio of 60% holding a Market Price of ₹ 110 per unit.
 Number of units issued by Goodluck Mutual Funds Limited are 1, 00,000 at an average cost of raising
funds of 10%.
Required:
a. What will be the value of the scheme as on 10th April, 2018?
b. Whether the value changes if valuation had been made on 9th April, 2018. - Explain
c. If values change what is the value of the scheme on 9th April, 2018?

Solution:

a. Valuation of the Equity oriented Mutual Fund Scheme on 10 th April, 2018


Units Last Traded: 8th February, 2018
Number of Days passed prior to valuation: 60
Since, 60 days have been passed since the last trade took place units are to be valued as a non-trade
scrip.
A proxy needs to be identified for computing the value of the scheme. Here in the problem Uncertain
Mutual Funds Limited having a market price of ₹ 110, and paying a dividend yield of 5.46% that means
paying a dividend of ₹ 110 × 5.46% = ₹ 6 per unit best fits the place of proxy.

Directorate of Studies, The Institute of Cost Accountants of India (Statutory Body under an Act of Parliament) Page 42
Work Book: Strategic Financial Management

If dividend of ` 6 is paid out at a Dividend Payout Ratio of 0.6, then Earnings per Unit is of ` 6 / 0.6 = ₹ 10.
Value of the Units on 10th April, 2018 of Goodluck Mutual Funds Limited will be ₹ 10 / 0.1 = ₹100
Value of the Mutual Fund Scheme: ₹ (100 × 1, 00,000 units) = ₹ 1, 00, 00,000

b. The value of units changes if valuation is made on 9 th April, 2018 (The previous day), As on 9th April, 2018, the
time span of 60 days not expires and therefore we can use the closing price of the units as listed in its
principal stock exchange for valuation purpose.
c. Value of the scheme on 9th April, 2018: ₹ (110.8 × 1,00,000 units) = ₹ 1,10,80,000

7. PQ Limited contemplating to issue shares at ₹120 each (Face Value of ₹100 each) bearing floatation costs of
5% on the issue price.
Expected return on capital employed is 20%, with an anticipated payment of dividend per share of ₹11.40.
MX Mutual Funds Limited investing in the same industry manages to yield a return similar to the expectation
of PQ Limited, bearing a floatation cost of 1.5% and management expenses (other than floatation costs) of
1.7% of yield.
Required:
a. Expected Market Capitalization of PQ Limited at the end of 1st year.
b. Investor’s expectation on returns of MX Mutual Funds Limited.

Solution:

a. Net sale proceeds from each share = ₹ 120 × (1 – 0.05) = ₹114


Return on Capital Employed = ₹114 × 20% = ₹22.80, out of which dividend payable ₹ 11.40
Cost of Equity Capital (Ke) = D1 / P0 = ₹ 11.40 / ₹ 120 = 9.5%
Expected Market Price per Equity Share at the end of 1st year = ₹ 22.80 / 9.5% = ₹240
Number of Equity shares to be issued = 1, 00,000
Expected Market Capitalization of PQ Limited at the end of 1st year = ₹ 240 × 1, 00,000 = ₹2, 40, 00,000

b. Investor Expectation on Returns from units of MX Mutual Funds Limited will be as follows:
Returns from Mutual Funds = (Investors‘ Expectation / 100 – Issue Expenses) + Annual Recurring Expenses
If returns from MX Mutual Funds Limited becomes equal to the expectation of PQ Limited regarding their
return on capital employed, since they both are in operations within the same sectors.
Then, Investors Expectation = {20% - (20% × 1.7%)} × (100 – 1.5) % = (19.66 × 0.985) = 19.3651%

8. Risk-Return Combinations relating to asset-mix of MFK and MFP are provided below.
MFK MFP
Securities Expected Return Total Risk Investment Expected Return Total Risk Investment (₹)
Equity Shares
Ambuja Cement - - - 15% 14% 800000
Tata Steel 12% 13% 1000000 - - -
Ashok Leyland 10% 15% 600000 - - -

Directorate of Studies, The Institute of Cost Accountants of India (Statutory Body under an Act of Parliament) Page 43
Work Book: Strategic Financial Management
Preferential Shares
J.K Tyre - - - 10% 8% 500000
Debentures
Essar Steel - - - 9% 8% 700000
ACC 8% 5% 400000 - - -

Correlation Matrix of companies


Company Name Tata Steel Ashok Leyland ACC Ambuja Cement Essar Steel J.K Tyre
Tata Steel 1 0.67 (0.4) - - -
Ashok Leyland 0.67 1 (0.5) - - -
ACC (0.4) (0.5) 1 - - -
Ambuja Cement - - - 1 (0.8) (0.6)
Essar Steel - - - (0.8) 1 0.7
J.K Tyre - - - (0.6) 0.7 1

Return on 91-day treasury bills is 7%.


Based on above information, Compute:
a. Expected Returns from units of MFK and MFP.
b. Risk Association with the units of MFK and MFP.
c. Advice where to commit funds and why?
d. Calculate Sharpe Ratio and establish a platform of performance appraisal.

Solution:

a. Expected Return from units of Mutual Fund E (R MF) = ∑ wi × Ri


Risk associated with returns of Mutual Funds (σ MF) = √ (wI2 × σI2) + (wJ2 × σJ2) + (wL2 × σL2) + 2 × wI × wJ × σI × σJ
× r(I,J) + 2 × wJ × wL × σJ × σL × r(J,L) + 2 × wI × wL × σI × σL × r(I,L)

Where, ‗I‘, ‗J‘, and ‗L‘ are the securities with which the mutual fund institution has constructed its portfolio.

Calculation showing Expected Return from Mutual Fund Investments


MFK MFP
Securities Companies
Returns (Ri) Weight (wi) E (RMF) Returns (Ri) Weight (wi) E (RMF)
Ambuja Cement - - - 15% 0.40 6%
Equity
Tata Steel 12% 0.5 6% - - -
Share
Ashok Leyland 10% 0.3 3% - - -
Pref. Share J.K Tyre - - - 10% 0.25 2.5%
Essar Steel - - - 9% 0.35 3.15%
Debentures
ACC 8% 0.2 1.6% - - -
 E (RMF) on MFK = (6% + 3% + 1.6%) = 10.6%
 E (RMF) on MFP = (6% + 2.5% + 3.15%) = 11.65%

Directorate of Studies, The Institute of Cost Accountants of India (Statutory Body under an Act of Parliament) Page 44
Work Book: Strategic Financial Management

b.
 Risk associated with returns of Mutual Funds ‗K‘ (σ MF) = √ (wT2 × σT2) + (wA2 × σA2) + (wA‘2 × σA‘2) + {2 × wT ×
wA × σT × σA × r(T,A)} + {2 × wA × wA‘ × σA × σA‘ × r(A,A‘)} + {2 × wT × wA‘ × σT × σA‘ × r(T,A‘)}
Or, (σMF) = √ (0.52 × 13%2) + (0.32 × 15%2) + (0.22 × 5%2) + (2 × 0.5 × 0.3 × 13% × 15% × 0.67) + (2 × 0.3 × 0.2 ×
15% × 5% × -0.5) + (2 × 0.5 × 0.2 × 13% × 5% × -0.4)
Or, (σMF) = 9.64%

 Risk associated with returns of Mutual Funds ‗P‘ (σ MF) = √ (w A2 × σA2) + (wE2 × σE2) + (wJ2 × σJ2) + {2 × wA ×
wE × σA × σE × r(A,E)} + {2 × wE × wJ × σE × σJ × r(E,J)} + {2 × wA × wJ × σA × σJ × r(A,J)}
Or, (σMF) = √ (0.42 × 14%2) + (0.352 × 8%) + (0.252 × 8%) + (2 × 0.4 × 0.35 × 14% × 8% × -0.8) + (2 × 0.35 × 0.25
× 8% × 8% × 0.7) + (2 × 0.4 × 0.25 × 14% × 8% × -0.6)
Or, (σMF) = 3.54%

c. It is very much clear from the above computations that MF P is providing higher returns (11.65%) on account
of a lower risk association (3.54%) than that of provided by MF K.
Therefore there is no reason to invest other than in MFP.

d. Sharpe Ratio: [{E (RMF) – RF} / σMF]


Parameters MFK MFP
Expected Return [E (RMF)] 10.6% 11.65%
Risk-free Rate of Return (RF) 7% 7%
Risk Premium {E (RMF) – RF} 3.6% 4.65%
Std. Deviation of Returns (σMF) 9.64% 3.54%
Sharpe Ratio 0.37 1.31
Rank 2ND 1ST

Result: The Sharpe ratio recommends investment in MF P because it provides a higher risk premium for each
unit of risk (Total Risk) association in comparison to MFK.

9. The following information is available of Mutual Fund A, Mutual B and Market Portfolio for the past six months:
Fund/Month (Return %) April 2017 May 2017 June 2017 July 2017 August 2017 September 2017
Fund A 3.00 1.75 (1.00) 3.50 1.50 0.00
Fund B 2.25 (1.25) 0.00 3.00 2.50 1.00
Market Portfolio 1.00 (0.75) 2.00 1.50 0.25 3.50

The 6 Month Treasury Bills carry an interest rate of 6% p.a. You are requested to evaluate performance of
Funds A, B and Market Portfolio under Morning Star Index.

Directorate of Studies, The Institute of Cost Accountants of India (Statutory Body under an Act of Parliament) Page 45
Work Book: Strategic Financial Management
Solution:

Computation of Factors
Month Fund A Fund B Market Portfolio
Return Risk of Loss Return Risk of Loss Return Risk of Loss
(1) (2) (3)=(2)-0.50* (4) (5)=(4)-0.50 (6) (7)=(6)-0.50
(If (2)<0.50 (If (4)<0.50 (If (6)<0.50
April 2017 3.00 0.00 2.25 0.00 1.00 0.00
May 2017 1.75 0.00 (1.25) 1.75 (0.75) 1.25
June 2017 (1.00) 1.50 0.00 0.50 2.00 0.00
July 2017 3.50 0.00 3.00 0.00 1.50 0.00
August 2017 1.50 0.00 2.50 0.00 0.25 0.25
September 2017 0.00 0.50 1.00 0.00 3.50 0.00
Total 8.75 2.00 7.50 2.25 7.50 1.50
Average 8.75/6=1.46 2.00/6=0.33 7.50/6=1.25 0.38 7.50/6=1.25 0.25
*Monthly Risk free return= 6%/12=0.50 p.m.

Computation of Morning Star Index (MSI)


Particulars Fund A (%) Fund B (%) Market Portfolio (%)
Average Monthly Return (I) 1.46 1.25 1.25
Average Monthly Risk of Loss (II) 0.33 0.38 0.25
Morning Star Index (MSI) (i.e. excess return) [(I)-(II)] 1.33 0.87 1.00
Ranking 1 2 3
Evaluation: Fund A has performed better than the market portfolio, while Fund B has not performed as good
as the market portfolio despite having the equivalent average return during the period.

10. A Mutual Fund Scheme having 400,000 units has shown NAV of ₹9.25 and ₹9.95 at the beginning and at the
end of the year respectively.
The Scheme has given two options:
(a) Pay ₹ 0.85 per unit as dividend and ₹ 0.70 per unit as capital gain, or
(b) These distributions are to be reinvested at an average NAV of ₹9.15 per unit.
You are required to find out what difference it would make in terms of return available and which option is
preferable?

Solution:

Particulars Value (₹)


Opening NAV 9.25
Closing NAV 9.95
Dividend 0.85
Capital Gain Appreciation [Closing NAV-Opening NAV] 0.70
Capital Gain Distribution 0.70
Price paid at the year beginning [400,000×₹9.25] 3,700,000

Directorate of Studies, The Institute of Cost Accountants of India (Statutory Body under an Act of Parliament) Page 46
Work Book: Strategic Financial Management
Option 1:

Returns are distributed to the mutual fund holders


Balance Sheet
Liabilities ₹ Assets ₹
NAV of Closing Date [₹9.95*400,000] 3,980,000 Fund Assets 4,600,000
Dividend Payable [₹ 0.85*400,000] 340,000
Capital Gain Distribution [₹ 0.70*400,000] 280,000
4,600,000 4,600,000

Returns = [Closing NAV-Opening NAV]/Opening NAV =[4,600,000-3,700,000]/ 3,700,000 = 24.324%

Option 2:

The distributions are reinvested at an average NAV of ₹9.15


Distributions reinvested
Particulars Value (₹)
Capital Gain [0.70×400,000] 280,000
Dividend [0.85×400,000] 340,000
Total distributions 620,000
No of Units [Total distributions/Average NAV PU] 620,000/9.15=67759.56 units 67759.56 units

Balance Sheet
Liabilities ₹ Assets ₹
NAV of Closing Date 4,600,000
-400,000 units 3,980,000 Fund Assets
-67759.56 units 620,000
4,600,000 4,600,000

Returns = [Closing NAV-Opening NAV]/Opening NAV =[4,600,000-3,700,000]/3,700,000 = 24.324%

Comment: Holding period return is the same from investor‘s view point irrespective of whether the return is
reinvested or distributed in the form of capital gains or dividends.

Directorate of Studies, The Institute of Cost Accountants of India (Statutory Body under an Act of Parliament) Page 47
Work Book: Strategic Financial Management
Study Note – 6

Capital Markets

1. Write a note on buy-back of shares by companies.

Answer:

The process of buy-back of shares was not permitted in India till 1988, which became permissible after
changes being done in the Companies Act 1956. Government of India and SEBI has issued certain
guidelines which are to be followed at the time of buy-back of shares. Buy back of shares is a corporate
financial strategy in which the shares of the company are bought by the company itself. Reliance, Bajaj,
and Ashok Leyland etc. are the few companies in India which have opted for buy-back of shares. There are
generally two methods that are applied in the corporate sector while buying-back of shares i.e. the tender
method or the open market purchase method. The company, under the tender method, offers to buy
back shares at a specific price during a specified period which is usually one month. Under the open
market purchase method, a company buys shares from the secondary market over a period of one year
subject to a maximum price fixed by the management. The open market purchase method is mostly
preferred by the companies due to the advantage of time and price flexibility.

The buy-back method has a huge impact on the P/E ratio of the company. The P/E ratio may rise if investors
view buyback positively or it may fall if the investors regard buyback negatively.

The advantages of Buy-back of shares are as follows:


(i) Efficient allocation of resources.
(ii) Ensuring price stability in share prices.
(iii) Tax advantages.
(iv) Exercising control over the company.
(v) Saving from hostile takeover.
(vi) Capital appreciation to investors which may otherwise be not available.

However, the share prices can also be manipulated by the promoters, speculators or collusive-traders
through the buy-back of shares, which can be counted as a disadvantage of this process.

2. Write a short note on depository participant.

Answer:

The securities such as shares, debentures, bonds, Government Securities, MF units etc. are now kept in
electronic form instead of physical form through the process of dematerialization. This speeds up the
process of sale, purchase and transmission of securities. The services of dematerialization, re-materialization,
transfer, sale etc. are provided by depositories registered under SEBI. The Depository Participant, thus, is an
agent of the depository which acts as an intermediary between the depository and the investors. The

Directorate of Studies, The Institute of Cost Accountants of India (Statutory Body under an Act of Parliament) Page 48
Work Book: Strategic Financial Management
investors can avail the depository related services by opening a Depository account, also known as Demat
a/c, with any of the Depository Participants. The shares and securities are converted into electronic form
and separate numbers are allotted to them. All the corporate benefits like bonus, stock splits, dividend etc
are also managed by the depositories and its agent i.e. Depository Participants.

3. What are the Advantages of a depository system?

Answer:

There are various advantages of the depository system to different stakeholders which are as follows:-

(I) For the Capital Market:


(i) It reduces risk of bad delivery;
(ii) It eliminates voluminous paper work and the time and money related with it;
(iii) It is time saving as it reduces settlement time and ensures quick settlement;
(iv) There is no odd lot problem of shares when the shares are kept with the depositories;
(v) It facilitates stock-lending and thus deepens the market.

(II) For the Investor:


(i) It reduces the risks associated with the loss or theft of documents and securities and eliminates
forgery;
(ii) It ensures liquidity as the process is automated and by speedy settlement of shares;
(iii) The depository holds the shares in electronic form so the investors are free from the physical holding
of shares;
(iv) It reduces costs such as stamp duty, transaction cost and brokerage; and
(v) It assists investors in securing loans against the securities.

(III) For the Corporate Sector or Issuers of Securities:


(i) It provides updated information of the shareholders and investors like names and addresses, etc.;
(ii) It builds up and enhances the image of the company;
(iii) It reduces the costs of the secretarial department;
(iv) It increases the efficiency of registrars and transfer agents; and
(v) It provides better facilities of communication with members.

4. What are the advantages of Optionally Convertible Debentures (OCDs)?

Answer:

Optionally Convertible Debentures (OCDs) are the debentures that include the option to get converted into
equity. The investor has the option to either convert these debentures into shares at price decided by the
issuer/agreed upon at the time of issue.

Advantages of OCD are:

Directorate of Studies, The Institute of Cost Accountants of India (Statutory Body under an Act of Parliament) Page 49
Work Book: Strategic Financial Management
(a) From Issuer
(i) Quasi-Equity: Dependence of Financial Institutions is reduced because of the inherent option for
conversion (i.e. since these are converted into equity, they need not be repaid in the near future.)
(ii) High Equity Line: It is possible to maintain Equity Price at a high level, by issuing odd-lot shares
consequent to conversion of the debentures, and hence lower floating stocks.
(iii) Dispensing Ownership: Optionally Convertible Debentures enable to achieve wide dispersal of
equity ownership in small lots pursuant to conversion.
(iv) Marketability: The marketability of the issue will become significantly easier, and issue expenses can
be expected to come down with the amounts raised becoming more.

(b) Investor
(i) Assured Interest: Investor gets assured interest during gestation periods of the project, and starts
receiving dividends once the project is functional and they choose to convert their debentures.
Thereby, it brings down the effective gestation period at the investor‘s end to zero.
(ii) Secured Investment: The investment is secured against the assets of the Company, as against
Company deposits which are unsecured.
(iii) Capital Gains: There is a possibility of Capital Gains associated with conversion, which compensates
for the lower interest rate on debentures.

(c) Government
(i) Debentures helped in mobilizing significant resources from the public and help in spreading the
Equity Investors, thereby reducing the pressure on Financial Institutions (which are managed by
Government) for their resources.
(ii) By making suitable tax amendments, benefits are extended to promote these instruments, to
safeguard the funds of Financial Institutions and encouraging more equity participation, which will
also require a higher compliance under Corporate Laws, whereby organisations can be monitored
more effectively.

5. Identify the aspects where credit rating unable to measure.

Answer:

Credit Rating do not measure the following-

i) Investment Recommendation: Credit Rating does not make any recommendation on whether to invest
or not.
ii) Investment Decision: They do not take into account the aspects that influence an investment decision.
iii) Issue Price: Credit Rating does not evaluate the reasonableness of the issue price, possibilities for capital
gains or liquidity in the secondary market.
iv) Risk of Prepayment: Ratings do not take into account the risk of prepayment by issuer, or interest or
exchange risks.
v) Statutory Compliance: Credit Rating does not imply that there is absolute compliance of statutory
requirements in relation to Audit, Taxation, etc. by-the issuing company.

Directorate of Studies, The Institute of Cost Accountants of India (Statutory Body under an Act of Parliament) Page 50
Work Book: Strategic Financial Management

Study Note – 7

Commodity Exchange

1. Briefly discuss the basic characteristics of Commodity Exchange in India.

Answer:

The basic characteristics of commodity exchange in India are:


(i) The units are inter-changeable and no value adding processes are performed on them. This allows the
units to be traded on exchanges without prior inspection.
(ii) Every commodity has a unique supply factor and as they are produced ―naturally‖.
(iii) Commodities are subject to cycles in demand from both intermediate players and end users. High
prices usually lead to a boost in resource investments causing excess supply in the future which
eventually pushes down commodity prices.
(iv) The commodities from different groups may be negatively correlated at a point of time. For example,
the prices of wheat and aluminum can move in the opposite direction as they are affected by a
different set of factors.
(v) There is a positive correlation between commodity prices and growth measures, although there may be
a significant lag between a pickup in industrial production and commodity prices.
(vi) A positive correlation is often seen between commodities and inflation indicators. In particular,
commodities tend to react to an early stage of inflation as raw material price appreciation generally
tends to precede, and quite often exceed consumer price inflation growth. While true over the very
long term, the relationship between inflation and commodity prices has been considerably weaker over
the last 10 years, which has been characterized by disinflation/low inflation.

The above characteristics may not be true for all commodities taken individually; however they are true for
diversified indices of industrial commodities and agricultural commodities.

2. List four benefits of commodity futures markets.

Answer:

(i) Price Discovery: Based on inputs regarding specific market information, the demand and supply
equilibrium, weather forecasts, expert views and comments, inflation rates, Government policies, market
dynamics, hopes and fears, buyers and sellers conduct trading at futures exchanges. This transforms into
continuous price discovery mechanism. The execution of trade between buyers and sellers leads to
assessment of fair value of a particular commodity that is immediately disseminated on the trading
terminal.
(ii) Price Risk Management: Hedging is the most common method of price risk management. It is strategy of
offering price risk that is inherent in spot market by taking an equal but opposite position in the futures
Directorate of Studies, The Institute of Cost Accountants of India (Statutory Body under an Act of Parliament) Page 51
Work Book: Strategic Financial Management
market. Futures markets are used as a mode by hedgers to protect their business from adverse price
change. Hedging benefits who are involved in trading of commodities like farmers, processors,
merchandisers, manufacturers, exporters, importers etc.
(iii) Improved product quality: The existence of warehouses for facilitating delivery with grading facilities
along with other related benefits provides a very strong reason to upgrade and enhance the quality of
the commodity to grade that is acceptable by the exchange. It ensures uniform standardization of
commodity trade, including the terms of quality standard: the quality certificates that are issued by the
exchange-certified warehouses have the potential to become the norm for physical trade.
(iv) Import- Export competitiveness: The exporters can hedge their price risk and improve their
competitiveness by making use of futures market. A majority of traders which are involved in physical
trade internationally intend to buy forwards. The purchases made from the physical market might
expose them to the risk of price risk resulting to losses. The existence of futures market would allow the
exporters to hedge their proposed purchase by temporarily substituting for actual purchase till the time
is ripe to buy in physical market. In the absence of futures market it will be meticulous, time consuming
and costly physical transactions.

3. Discuss the role of Forward Markets Commission.

Answer:

Forward Markets Commission provides regulatory oversight in order to ensure financial integrity (i.e. to
prevent systematic risk of default by one major operator or group of operators), market integrity (i.e. to
ensure that futures prices are truly aligned with the prospective demand and supply conditions) and to
protect and promote interest of customers/ non-members. It prescribes the following regulatory measures:

(i) Limit on net open position as on the close of the trading hours. Sometimes limit is also imposed on intra-
day net open position. The limit is imposed operator-wise and in some cases, also member- wise.
(ii) Circuit-filters or limit on price fluctuations to allow cooling of market in the event of abrupt upswing or
downswing in prices.
(iii) Special margin deposit to be collected on outstanding purchases or sales when price moves up or
down sharply above or below the previous day closing price. By making further purchases/sales
relatively costly, the price rise or fall is sobered down. This measure is imposed only on the request of the
exchange.

4. Write a short note on Commodity Exchange.

Answer:

A commodities exchange is an exchange where various commodities and derivatives products are traded.
Most commodity markets across the world trade in agricultural products and other raw materials (like
wheat, barley, sugar, maize, cotton, cocoa, coffee, milk products, pork bellies, oil, metals, etc.) and
contracts based on them. These contracts can include spot, forwards, futures and options on futures. Other

Directorate of Studies, The Institute of Cost Accountants of India (Statutory Body under an Act of Parliament) Page 52
Work Book: Strategic Financial Management
sophisticated products may include interest rates, environmental instruments, swaps, or ocean freight
contracts.

Commodities exchanges usually trade futures contracts on commodities, such as trading contracts to
receive something, say corn, in a certain month. A farmer raising corn can sell a future contract on his corn,
which will not be harvested for several months, and guarantee the price he will be paid when he delivers; a
breakfast cereal producer buys the contract now and guarantees the price will not go up when it is
delivered. This protects the farmer from price drops and the buyer from price rises.

Speculators and investors also buy and sell the futures contracts in attempt to make a profit and provide
liquidity to the system. However, due to the financial leverage provided to traders by the exchange,
commodity futures traders face a substantial risk.

A commodity exchange provides the rules, procedures, and physical for commodity trading, oversees
trading practices, and gathers and disseminates marketplace information. Commodity exchange
transactions take lace on the commodity exchange floor, in what is called a pit, and must be effected
within certain time limits.

Directorate of Studies, The Institute of Cost Accountants of India (Statutory Body under an Act of Parliament) Page 53
Work Book: Strategic Financial Management

Study Note – 8

Security Analysis and Portfolio Management

1. Choose the correct alternative:

(i) An investor owns a stock portfolio consisting of four stocks. He invested in stock 20% in stock A; 25% in
stock B; 30% in stock C and 25% in stock D. The betas of these four portfolios are :0.9; 1.3; 1.2 and 1.7
respectively. The beta of portfolio is-
a) 1.12
b) 1.29
c) 1.45
d) 1.76

(ii) Security Market Line (SML) shows the relationship between return on the stock and
a) Return on the market portfolio
b) Risk-free rate of return
c) Beta of the stock
d) Variance of the stock returns

(iii) Historically, when the market return changed by 10%, the return on the stock of A Ltd. changed by 16%.
If the variance of the market return is 257.81, what would be the systematic risk for A Ltd.?
a) 320%
b) 480%
c) 660%
d) 720%

(iv) The intercept of the Security Market Line (SML) is:


a) E(Rm) – Rf
b) 1/(E(Rm) - Rf)
c) Rf - E(Rm)
d) Rf

(v) Residual analysis is a test of


a) Weak-form of market efficiency
b) Semi-strong form of market efficiency
c) Strong form of market efficiency
d) Super-strong form of market efficiency

(vi) Securities A and B have a standard deviation of 10% and 15% respectively. The respective average
returns are 12% and 20%. Investor X has limited funds. Which is safer security for investment?
Directorate of Studies, The Institute of Cost Accountants of India (Statutory Body under an Act of Parliament) Page 54
Work Book: Strategic Financial Management
a) A is more secured.
b) B is more secured.
c) Both A & B are equally secured.
d) Incomplete information.

(vii)Which of the following is on the horizontal axis of the Security Market Line?
a) Beta
b) Standard deviation
c) Expected return
d) Required return

(viii) The beta of stock of A Ltd. is 2.00 and is currently in equilibrium. The required rate of return on the stock
is 12% and expected return on the stock is 10%. Suddenly, due to change in the economic conditions,
the expected return on the market increases to 12%. Other things remaining the same, what would be
new required rate of return on the stock?
a) 15.0%
b) 16.0%
c) 20.0%
d) 22.5%

Solution:

(i) b (iii) c (v) b (vii) a


(ii) c (iv) d (vi) b (viii) b

2. a) Calculate the expected rate of return for each security from the figures below.
Security A B C D E
β – values 1.3 1.6 1 1.5 0
Actual Return (Rp) 21 23.6 16 20 4.8
Investments (In ` Lacks.) 1.25 1.5 1.25 1.45 0.8

b) Further if Mr. Anup Ahuja wants to form a portfolio with the above investible funds in respective securities
what would be his expected return on such asset-mix?

The following data is available for three securities:


Security A B C
Beta coefficient 1.4 1.5 1.6
Standard Deviation of Market Return 0.5 0.5 0.5
Total Risk 0.6 0.8 0.85

For which of these securities the systematic component explains the largest share of total risk?

Directorate of Studies, The Institute of Cost Accountants of India (Statutory Body under an Act of Parliament) Page 55
Work Book: Strategic Financial Management
Solution:

a) Since Value of Beta (β) for security C is 1, the market rate of return is equal to the return as reflected by
security C (i.e. 16%).
Similarly the risk free rate of return is equal to the return as provided by security E (i.e. 4.8%).
Therefore, Risk premium awarded by market is equal to (RM-RF), i.e. (16-4.8) % or, 11.2%.

Expected Rate of Return E (Ri) = RF + βi (RM-RF)


Security A B C D E
Risk free Rate of Return (RF) 4.8% 4.8% 4.8% 4.8% 4.8%
Beta Values (βi) 1.3 1.6 1 1.5 0
Risk premium from Market (RM - RF) 11.2% 11.2% 11.2% 11.2% 11.2%
Effective Risk Premium [βi (RM - RF)] 14.56% 17.92% 11.2% 16.8% 0%
Expected Rate of Return E (Ri) 19.36% 22.72% 16% 21.6% 4.8%

b) Computation of Weighted Beta


Security Investible Funds (In `) Weightage of Investment Beta Values (βi) Weighted Beta
A 1,25,000 0.2 1.3 0.26
B 1,50,000 0.24 1.6 0.384
C 1,25,000 0.2 1 0.2
D 1,45,000 0.232 1.5 0.348
E 80,000 0.128 0 0
Total 6,25,000 1 1.192

In computation of the expected return from portfolio of the above securities weighted beta will be used as
a proxy to evaluate effective risk premium.
Expected Rate of Return from portfolio E (Rp) = RF + β (RM – RF).
Where, β = Weighted Beta.
E (Rp) = 4.8% + 1.192 (11.2%) = 4.8% + 13.3504% = 18.15%.
Total Risk = Systematic Risk + Un-systematic Risk
Var. (Rit) = βi2 σM2 + Var. (eit)

Table showing computation of Systematic and Un-systematic Risk


Securities A B C
Beta Values (β) 1.4 1.5 1.6
Square of Beta Values (β2) 1.96 2.25 2.56
Std. Deviation of Market Return (σ M) 0.5 0.5 0.5
Variance of Market Return (σM2) 0.25 0.25 0.25
Systematic Risk (βi2× σM2) 0.49 0.5625 0.64
Total Risk [Var. (Rit)] 0.6 0.8 0.85
Unsystematic Risk [Var. (eit)] 0.11 0.2375 0.21
Systematic Risk to Total Risk Ratio [(βi2× σM2)/ Var. (Rit)] 0.8167 0.7031 0.7529

Directorate of Studies, The Institute of Cost Accountants of India (Statutory Body under an Act of Parliament) Page 56
Work Book: Strategic Financial Management

Therefore, among all the securities the largest share of systematic risk explained out of total risk is been
followed in case of security A.

3. Mr. Abinash has invested his fund in two securities of the same company, details of which has been laid in
the following section.
Security Equity Share Preference Share
Extent of Commitment 0.4 0.6
Expected Return 14% 11%
Volatility of Returns (Measured in terms of Sigma) 12% 15%
Covariance 120
Compute:
a) Expected Return from the Asset-mix.
b) Risk associated with the portfolio.
c) Minimum commitment of funds in securities to mitigate the exposure to risks at the given value of
correlation coefficient.
Should the investor try to have a different combination of securities?

Solution:

a) Expected Return from the Asset-Mix [E (RP)] = ∑wi × Ri.


Where, ‗i‘ runs from 1 to ‗n‘, ‗n‘ is the number of securities in the portfolio.
Expected Return from portfolio [E (RP)] = {(0.4 × 14%) + (0.6 × 11%)} = 12.2%.

b) Measure of Portfolio Risk (σP) = √ (wE2 × σE2) + (wP‘2 × σP‘2) + 2 × wE × wP‘ × σE × σP‘ × r(E,P‘)
Where, wE = Investment in Equity Share, w P = Investment in Preferential Share, σE = Volatility in Equity Returns,
σP‘ = Volatility in Preferential Returns, r (E,P‘) = Correlation Coefficient between Equity Share and Preferential
Share.
Correlation Coefficient (r(E,P‘)) = Cov. (E,P‘)/ σE × σP‘ = 120 / (12 × 15) = 0.67
Portfolio Risk (σP) = √ {(0.4)2 × (12%) 2} + {(0.6)2 × (15%) 2} + {2 × (0.4 × 0.6) × (12 × 15 × 0.67)} = 12.73%.

c) Minimum Investment required to mitigate risks in Equity (w E (Min.)) and in Preference Shares (wP‘ (Min.))
wE (Min.) = σP‘2 – {σE × σP‘ × r(E,P‘)} / σE2 + σP‘2 – (2 × σE × σP‘ × r(E,P‘))
wE (Min.) = (15%)2 – (12 × 15 × 0.67) / (12%)2 + (15%)2 – (2 × 12 × 15 × 0.67) = 81.69%.
wP‘ (Min.) = (1 - wE (Min.)) = (1 – 0.8169) = 18.31%.

Mr. Abinash is expected to invest his available funds in Equity Share @ 81.69% and in Preferential Share @ 18.31%
to make his portfolio bearing with minimum risk. At present his portfolio bears a combination of 40% and 60% on
equity and preferential share investments respectively, which is not the optimum one. Therefore he is advised to
try a different combination of securities.

Directorate of Studies, The Institute of Cost Accountants of India (Statutory Body under an Act of Parliament) Page 57
Work Book: Strategic Financial Management
4. Return on asset mix of Sambhavana Limited and Bhavana Limited at different stages of economy with
respective chances of taking place is provided in the following.
State of Economy Chance of Occurrence Sambhavana Ltd. Bhavana Ltd.
Boom 0.1 20% 26%
Growth 0.5 15% 18%
Decline 0.3 10% 8%
Depression 0.1 5% 0%
You are required to compute the following.
a) Expected Return of the companies mentioned above.
b) Volatility in expected return of the companies expressed in terms of Standard Deviation.
c) The co-movement and the degree of association between returns.

As an Investment Advisor in which of the above companies you would recommend to invest funds –
Establish a ground supporting your recommendation.

Solution:

a) Expected Return on Investment [E (Ri)] = ∑pi × Ri , where, ‗i‘ runs from ‗1‘ to ‗n‘.
 Expected Return from Sambhavana Limited = {(20% × 0.1) + (15% × 0.5) + (10% × 0.3) + (5% × 0.1)} = 13%,
 Expected Return from Bhavana Limited = {(26% × 0.1) + (18% × 0.5) + (8% × 0.3) + (0% × 0.1)} = 14%.

b) Volatility in expected return of the companies (σ) = √ ∑p i × {Ri – E (Ri)} 2

Table showing computation of variation in securities of Sambhavana Limited


Probability (pi) R S‘ E (Ri) Rs‘ - E (Ri) [Rs‘ - E (Ri)]2 pi × {RS‘ – E (Ri)} 2
0.1 20% 13% 7% 49% 4.9%
0.5 15% 13% 2% 4% 2%
0.3 10% 13% -3% 9% 2.7%
0.1 5% 13% -8% 64% 6.4%

Return and Risk from securities of Sambhavana Limited is denoted as RS‘ and σS‘ respectively.
(σS‘) = √ (4.9% + 2% + 2.7% + 6.4%) = 4%

Table showing computation of variation in securities of Bhavana Limited


Probability (pi) RB E (Ri) RB - E (Ri) [RB - E (Ri)]2 pi × {RB – E (Ri)} 2
0.1 26% 14% 12% 144% 14.4%
0.5 18% 14% 4% 16% 8%
0.3 8% 14% -6% 36% 10.8%
0.1 0% 14% -14% 196% 19.6%

Return and Risk from securities of Bhavana Limited is denoted as R B‘ and σB respectively.
σB = √ (14.4% + 8% + 10.8% + 19.6%) = 7.27%.

Directorate of Studies, The Institute of Cost Accountants of India (Statutory Body under an Act of Parliament) Page 58
Work Book: Strategic Financial Management
c) Co-movements can be studied between securities by computing the Covariance and the association can
be studied using correlation coefficient between securities.
Covariance between securities Cov. (S‘, B) = ∑ pi × {Rs‘ - E (Ri)} × {RB - E (Ri)}
Or, Cov. (S‘, B) = (8.4% + 4% + 5.4% + 11.2%) = 29%

Table showing computation of Covariance between securities


Probability (pi) {Rs‘ - E (Ri)} {RB - E (Ri)} pi × {Rs‘ - E (Ri)} × {RB - E (Ri)}
0.1 7% 12% 8.4%
0.5 2% 4% 4%
0.3 -3% -6% 5.4%
0.1 -8% -14% 11.2%

Value of correlation coefficient [r (S‘, B)] = Cov. (S‘, B) / σS‘ × σB = 29% / 4% × 7.27% = 0.9972

As both the risk and return is high in case of Bhavana Limited we need to use the Coefficient of Variation to
determine where to invest funds.

Coefficient of Variation = Expected Return / Std. Deviation of Returns

It basically replicates the return from securities for each degree of risk taken. Lower the proportion higher the
eligibility of a security to qualify for investment.

Company Name Sambhavana Limited Bhavana Limited


Coefficient of Variation 3.25 1.93

It is clear from the above measure that investment in Bhavana Limited is better than to invest in
Sambhavana Limited. It is thereby recommended to invest in securities of Bhavana Limited.

5. Following information is available in relation to the companies under pharmaceutical industry.


Company Name Cipla Ranbaxy
Expected Return 12% 18%
Standard Deviation of Returns 0.16 0.24

Construct portfolios with the following weights and determine the expected return on each of such asset-mix
along with the degree of risk association with the respective returns, when value of correlation coefficient
between securities are exactly 1, -1, 0, and 0.5.

Cipla 1 0.8 0.6 0.4 0.2 0


Ranbaxy 0 0.2 0.4 0.6 0.8 1
Total Weight 1 1 1 1 1 1

Directorate of Studies, The Institute of Cost Accountants of India (Statutory Body under an Act of Parliament) Page 59
Work Book: Strategic Financial Management

Solution:

Required Formulas:
Expected Return from a Portfolio [E (RP)] = ∑ wi × Ri
Expected Risk from a Portfolio (σP) = √ (wI2 × σI2) + (wJ2 × σJ2) + 2 × wI × wJ × σI × σJ × r(I,J)
Where, -1 < r(I,J) < 1, and ‗I‘ and ‗J‘ are two securities.

And when r(I,J) = ± 1, then we use the following equations which are derived form of the above one.
 Expected Risk from a Portfolio (σP) = {(wI × σI) + (wJ × σJ)}, when, r = +1,
 Expected Risk from a Portfolio (σP) = {(wI × σI) - (wJ × σJ)}, when, r = -1.

Using the formulas stated above we find the following table as result of the required questions regarding
Expected Return from a Portfolio and Expected Risk from the same Portfolio at different combinations of
investible funds and at different degrees of associations between securities.

Combinations of Weight Portfolio Return [E Portfolio Risk (σP)


Cipla Ranbaxy (RP)] r = +1 r = -1 r=0 r = 0.5
1 0 12% 16% 16% 16% 16%
0.8 0.2 13.2% 17.6% 8% 13.67% 15.76%
0.6 0.4 14.4% 19.2% 0% 13.58% 16.63%
0.4 0.6 15.6% 20.8% 8% 15.76% 18.45%
0.2 0.8 16.8% 22.4% 16% 19.46% 20.98%
0 1 18% 24% 24% 24% 24%

6. A Portfolio Manager has the following four stocks in his portfolio:


Security No of shares Market Price (₹)per share β = Beta
A 12,000 40 0.9
B 6,000 20 1.0
C 10,000 25 1.5
D 2,000 225 1.2

Compute the following:


a) Portfolio Beta (β)
b) If the Portfolio Manager seeks to reduce the Beta to 0.8, how much risk-free investment should he bring
in? Verify the result.

Directorate of Studies, The Institute of Cost Accountants of India (Statutory Body under an Act of Parliament) Page 60
Work Book: Strategic Financial Management
Solution:

a) Portfolio Beta

Security No of Shares Market price per share Value Amount (₹) % of Total Amount Beta Weighted Beta
A 12,000 40 4,80,000 0.3692 0.9 0.3323
B 6,000 20 1,20,000 0.0923 1.0 0.0923
C 10,000 25 2,50,000 0.1923 1.5 0.2885
D 2,000 225 4,50,000 0.3462 1.2 0.4154
13,00,000 1.000 1.129

Hence Portfolio Beta (β) = 1.129

b) Required Beta= 0.8


It should become 0.8/1.129=70.86% of the present portfolio.
If ₹ 13,00,000 is 70.86%
13,00,000×100
Total Portfolio should be = =₹18,34,600
70.86%
Additional investment in zero risk should be= ₹ (18,34,600-13,00,000)= ₹5,34,600

Revised Portfolio will be:


Security No of Shares Market price per share Value Amount (₹) % of Total Amount Beta Weighted Beta
A 12,000 40 4,80,000 0.2616 0.9 0.2354
B 6,000 20 1,20,000 0.0654 1.0 0.0654
C 10,000 25 2,50,000 0.1363 1.5 0.2045
D 2,000 225 4,50,000 0.2453 1.2 0.2944
Risk Free Asset 53,460 10 5,34,600 0.2914 0 0
18,34,000 1.000 0.7997 or 0.80

7. (a) An investor estimates return on shares in two different companies under four different scenarios as under:
Scenario Probability of its happening Return on Security A (%) Return on Security B (%)
I 0.2 12 10
II 0.4 16 20
III 0.3 18 25
IV 0.1 25 30

You are required to:


(i) Calculate Expected rate of return if the investor invests all his funds in Security A alone or in Security
B alone.
(ii) Determine the preferred security based on return.
(iii) Ascertain the risk associated with each of the security.

Directorate of Studies, The Institute of Cost Accountants of India (Statutory Body under an Act of Parliament) Page 61
Work Book: Strategic Financial Management
(v) If the investor invests 40% in Security A and 60% in Security B, what is the expected return and the
associated risk.
Solution:

(i) and(iii) Expected Returns and Risks Associated of Securities A and B

Expected Return and Risk of Security A


Scenario Probability (P) Return (%) Expected Return (%) = Deviation (D) from Deviation square (D2) Variance
mean
(1) (2) (3) (4)= (2)*(3) 5=(3)- Σ (4)] (6)=[(3)- Σ (4)]2 6=(2)*(5)
I 0.2 12 2.4 -4.7 22.09 4.418
II 0.4 16 6.4 -0.7 0.49 0.196
III 0.3 18 5.4 1.3 1.69 0.507
IV 0.1 25 2.5 8.3 68.89 6.889
16.7 12.01

Expected return on Security A=16.7%


Risk on Security A (σ) = Varaince= 12.01=3.465

Expected Return and Risk of Security B


Scenario Probability (P) Return (%) Expected Return (%) = Deviation (D) from Deviation square (D2)Variance
mean
(1) (2) (3) (4)= (2)*(3) 5=(3)- Σ (4)] (5)=[(3)- Σ (4)]2 6=(2)*(5)
I 0.2 10 2.0 -10.5 110.25 22.05
II 0.4 20 8.0 -0.5 0.25 0.1
III 0.3 25 7.5 4.5 20.25 6.075
IV 0.1 30 3.0 9.5 90.25 9.025
20.5 37.25

Expected return on Security A=20.5%


Risk on Security A (σ) = Varaince= 37.25=6.103

(ii) Expected return of Security B is higher than the Security A. So the investor will prefer Security B in terms of
Return.

(iv) Expected Return and Risk of Portfolio


Computation of Covariance of Securities A and B
Scenario Probability Deviation from Mean for A Deviation from Mean for B (%) Deviation Covariance
(P) (%) Product
(1) (2) (3) (4) (5) (6)=(2)*(5)
I 0.2 -4.7 -10.5 49.35 9.87

Directorate of Studies, The Institute of Cost Accountants of India (Statutory Body under an Act of Parliament) Page 62
Work Book: Strategic Financial Management
II 0.4 -0.7 -0.5 0.35 0.14
III 0.3 1.3 4.5 5.85 1.755
IV 0.1 8.3 9.5 78.85 7.885
19.65

Covariance of Securities A and B = 19.65

Correlation coefficient of Securities A and B= Cov (A and B)÷(S.D. of A and S.D of B)


=19.65/(3.465*6.103)=0.9292

Risk of portfolio i.e. standard deviation of Portfolio A and B [40% and 60% Ratio]
(σ) A and B=
3.4652 × 0.402 + 6.1032 × 0.602 + 2 × 0.9292 3.465 × 0.40 × 6.103 × 0.60

= 1.921 + 13.408 + 9.431= 24.76 = 4.975

Return = 40% of Return of Security A + 60% of Security B


= 0.40×16.7%+0.60×20.5%= 18.98%

8. Two securities X and Y have standard deviations of 4% and 10%. An investor is having a surplus of ₹10 Lakh
for investment in these two securities. How much should he invest in each of these securities to minimize risk,
if the correlation coefficient for X and Y is — (a) -1; (b) -0.40; (c) 0

Answer:

If rXY is CovXY is Computation Investment


-1 (-1*4*10)=-40 Proportion of Investment in Security X, Wx =[102-(-40)]/[42+102-2(-40)] .714X
Proportion of Investment in Security X, Wy=(1-.714)Y .286Y
Investment in X of `10 lakh 714000
Investment in Y of `10 lakh 286000
-.40 (-.4*4*10)=-16 Proportion of Investment in Security X, Wx =[102-(-16)]/[42+102-2(-16)] .784X
Proportion of Investment in Security X, Wy=(1-.714)Y .216Y
Investment in X of `10 lakh 784000

Directorate of Studies, The Institute of Cost Accountants of India (Statutory Body under an Act of Parliament) Page 63
Work Book: Strategic Financial Management
Investment in Y of `10 lakh 216000
0 (0*4*10)=0 Proportion of Investment in Security X, Wx =[102-0)]/[42+102-2(0)] .862X
Proportion of Investment in Security X, Wy=(1-.714)Y .138Y
Investment in X of `10 lakh 862000
Investment in Y of `10 lakh 128000

9. As an investment manager, you are given the following information:


Investment Initial price (₹) Dividend Market price Beta
Equity shares of
A Ltd. 70 5 140 0.8
B Ltd. 80 5 150 0.7
C Ltd. 90 5 270 0.5
Govt. of India Bonds 1000 160 1010 0.95

Risk free return may be taken at 16%.


You are required to calculate:
a) Expected rate of return of portfolio using CAPM.
b) Average return of Portfolio.

Solution:

Calculation of expected rate of return of Portfolio


Investment Amount Market price Capital Gain Dividend Total
Equity shares of
A 70 140 70 5 75
B 80 150 70 5 75
C 90 270 180 5 185
Govt. of India Bond 1000 1010 10 160 170
Total 1240 1570 330 175 505

a) Expected rate of return on portfolio =(505/1240)×100=40.73%


CAPM Model
E[RP]=RM+β(RM-RF)
A Ltd. =16+0.8(40.73-16)=35.78%
B Ltd. = 16+0.7(40.73-16)=33.31%
C Ltd. = 16+0.5(40.73-16)=28.37%
Govt. of India Bonds= 16+0.95(40.73-16)=39.49%

b) Simple average return of portfolio=935.78+33.31+28.37+39.49)/4=34.24%


Average of Beta=(0.80+0.70+0.50+0.95)/4=0.7375

Directorate of Studies, The Institute of Cost Accountants of India (Statutory Body under an Act of Parliament) Page 64
Work Book: Strategic Financial Management

Alternative Approach for Average Return:

Weighted Average Return


Securities Cost Proportion Expected Return Weighted Return (%)
A 70 0.056 35.78 2.004
B 80 0.065 33.31 2.132
C 90 0.073 28.37 2.043
Govt. of India Bonds 1,000 0.806 39.49 31.829
1,240 1,000 38.008

10. An investor has two portfolios known to be on minimum variance set for a population of three securities A, B
and C below mentioned weights —
WA WB WC
Portfolio X 0.30 0.40 0.30
Portfolio X 0.20 0.50 0.30

It is supposed that there are no restrictions on short sales.


(a) What would be the weight for each stock for a portfolio constructed by investing ₹6,00000 in Portfolio X
and ₹4,00000 in Portfolio Y?
(b) Suppose the investor invests ₹5,00000 out of ₹10,00000 in Security A. How he will allocate the balance
between security B and C to ensure that his portfolio is on minimum variance set?

Solution:

a) Investment in Individual Securities


Security Portfolio X Portfolio Y Total Weight
A 6,00000 x 0.30 = 1,80000 4,00000 x 0.20 = 80000 2,60000 2,60000 ÷10,00000 = 0.26
B 6,00000 x 0.40 = 2,40000 4,00000 x 0.50 = 2,00000 4,40000 4,40000 ÷10,00000= 0.44
C 6,00000 x 0.30 = 1,80000 4,00000 x 0.30 = 1,20000 3,00000 3,00000 ÷10,00000 = 0.30
6,00000 4,00000 10,00000 1.0000

b) Investment Strategy to Ensure Minimum Variance


Given the following equations WA = 0.50 (₹5,00000 ÷ ₹10,00000)
WA + WB + WC =1
Therefore it naturally follows that
WB + WC = 0.50 ...(1)

A simple linear equation establishing an equation between two variables WA and WB or the Variables WB
and WC in the given manner—
WC = a + b WB

Directorate of Studies, The Institute of Cost Accountants of India (Statutory Body under an Act of Parliament) Page 65
Work Book: Strategic Financial Management

Substituting the values of WA & WB from the data given (Portfolio X and Y), we get -
0.30 = a + b x 0.40
0.30 = a + b x 0.50
b=0
a =0.30
WC = 0.30 - WB
or
WC + 0 WB = 0.30 ...(2)

Therefore solving (1) and (2) we get WC = 0.30 and WB = 0.20

Conclusion: Allocation of Funds -


A = ₹ 5,00000 (Given)
B= 0.20 x ₹10,00000=₹ 2,00000
C= 0.30 x ₹ 10,00000 = ₹3,00000

Alternatively,
Since the Proportion of Investment in C is 0.30 and is constant across both the Portfolio, any linear equation
drawn from the Data given would result in the Weight of C being a constant 0.30.

Therefore WA= 0.50 (Given), WC = 0.30 (Constant), therefore WB = 0.20 (WB= 1- 0.50 - 0.30 = 0.20).

Directorate of Studies, The Institute of Cost Accountants of India (Statutory Body under an Act of Parliament) Page 66
Work Book: Strategic Financial Management

Study Note – 9

Financial Risks

1. How to assess and mitigate Market Risk?

Answer:

The potential loss amount due to market risk may be measured in a number of ways or conventions.
Traditionally, one convention is to use Value at Risk (VaR). The conventions of using Value at risk are well
established and accepted in the short-term risk management practice. However, it contains a number of
limiting assumptions that constrain its accuracy. The first assumption is that the composition of the portfolio
measured remains unchanged over the specified period. Over short time horizons, this limiting assumption is
often regarded as reasonable. However, over longer time horizons, many of the positions in the portfolio may
have been changed. The Value at Risk of the unchanged portfolio is no longer relevant.
 Market risk cannot be eliminated through diversification, though it can be hedged against.
 Financial risk, market risk, and even inflation risk, can at least partially be moderated by diversification.
 The returns from different assets are highly unlikely to be perfectly correlated and the correlation may
sometimes be negative. However, share prices are driven by many factors, such as the general health of
the economy which will increase the correlation and reduce the benefit of diversification.
 If one constructs a portfolio by including a wide variety of equities, it will tend to exhibit the same risk and
return characteristics as the market as a whole, which many investors see as an attractive prospect.
 However, history shows that even over substantial periods of time there is a wide range of returns that an
index fund may experience; so an index fund by itself is not ―fully diversified‖. Greater diversification can be
obtained by diversifying across asset classes; for instance a portfolio of many bonds and many equities can
be constructed in order to further narrow the dispersion of possible portfolio outcomes.

2. Discuss the different types of Credit Risks.

Answer:

Credit risk can be classified in the following way:


(i) Credit default risk: The risk of loss arising from a debtor being unlikely to pay its loan obligations in full or the
debtor is more than 90 days past due on any material credit obligation; default risk may impact all credit
sensitive transactions, including loans, securities and derivatives.
(ii) Counterparty risk: The risk of loss arising from non performance of counterparty in trading activities such as
buying and selling of commodities, securities, derivatives and foreign exchange transactions. If inability to
perform contractual obligations in such trading activities is communicated before the settlement date of
the transaction, then counterparty risk is in the form of pre-settlement risk, while if one of the counterparty
defaults on its obligations on the settlement date, the counterparty risk is in the form of settlement risk.

Directorate of Studies, The Institute of Cost Accountants of India (Statutory Body under an Act of Parliament) Page 67
Work Book: Strategic Financial Management
(iii) Concentration risk: The risk associated with any single exposure or group of exposures with the potential to
produce large enough losses to threaten a lender‘s core operations. It may arise in the form of single name
concentration or industry concentration.
(iv) Country risk: The risk of loss arising from sovereign state freezing foreign currency payments (transfer/
conversion risk) or when it defaults on its obligations (sovereign risk).

3. What are the causes of liquidity risk?

Answer:

Market liquidity risk arises from situations in which a party interested in trading an asset cannot do it because
nobody in the market wants to trade for that asset. Liquidity risk becomes particularly important to parties who
are about to hold or currently hold an asset, since it affects their ability to trade. Manifestation of liquidity risk is
very different from a drop of price to zero. The important causes of liquidity risk are:
(i) In case of a drop of an asset‘s price to zero, the market is saying that the asset is worthless. However, if one
party cannot find another party interested in trading the asset, this can potentially be only a problem of the
market participants with finding each other. This is why liquidity risk is usually found to be higher in emerging
markets or low-volume, less-structured markets.
(ii) On the other hand, funding liquidity risk is a financial risk due to uncertain liquidity. An institution might lose
liquidity if its credit rating falls, it experiences sudden unexpected cash outflows, or some other event causes
counterparties to avoid trading with or lending to the institution. A firm is also exposed to liquidity risk if
markets on which it depends are subject to loss of liquidity.

4. Write a short note on Asset-Backed Risk.

Answer:

It is the risk that the changes in values of one or more assets that support an asset-backed security will
significantly impact the value of the supported security. This kind of risk especially arises in securitization
transactions whereby cash flows due on assets/receivables are pooled together to issue securities, the servicing
of which is backed by the cash flows on such underlying assets. The factors that may cause changes in values
of assets backing the securities include interest rate, term modification, and prepayment risk.

Prepayment Risk: Prepayment is the event that a borrower prepays the loan prior to the scheduled repayment
date. Prepayment takes place when the borrowers can benefit from it, for example, when the borrowers can
refinance the loan at a lower interest rate from another lender. Prepayments result in loss of future interest
collections because the loan is paid back pre-maturely and can be harmful to the loan-backed securities,
especially for long term securities. A second, and maybe more important consequence of prepayments, is the
impudence of un-scheduled prepayment of principal that will be distributed among the securities according to
the priority of payments, reducing the outstanding principal amount, and thereby affecting their weighted
average life.

Directorate of Studies, The Institute of Cost Accountants of India (Statutory Body under an Act of Parliament) Page 68
Work Book: Strategic Financial Management
If an investor is concerned about a shortening of the term about contraction risk and the opposite would be the
extension risk, the risk that the weighted average life of the security is extended. In some circumstances, it will
be borrowers with good credit quality that prepay and the credit quality pool backing securities will deteriorate
as a result. Other circumstances will lead to the opposite situation.

Directorate of Studies, The Institute of Cost Accountants of India (Statutory Body under an Act of Parliament) Page 69
Work Book: Strategic Financial Management
Study Note – 10

Financial Derivatives – Instruments for Risk Management

1. Choose the correct alternative

(i) An investor bought 2,000 shares of X Ltd. for ₹90 per share. The initial margin is 50%. The maintenance
margin is 40%. If the stock price decreases to ₹ 70 per share. The additional funds put by the investors to
his margin account is –
a) ₹ 20,000
b) ₹ 20,500
c) ₹ 21,000
d) ₹ 22,000

(ii) An investor purchases a September Put Option of Y Ltd. with a strike price of ₹100 for a premium of ₹ 6.
Till what level the investor will not realize his profit?
a) ₹ 90
b) ₹ 92
c) ₹94
d) ₹96

(iii) An investor purchases a July Call Option of X Ltd. with a strike price of `100 for a premium of `7. Till what
level the investor will not realize his profit.
a) ₹ 105
b) ₹ 107
c) ₹110
d) ₹115

(iv) In a put-call parity, the pay-offs of buying stock can be replicated by:
a) Buying a call and buying a put option
b) Buying a call and writing a put option
c) Writing a call and buying a put option
d) Writing a call and writing a put option

(v) A stock is currently sells at ₹350. The put option to sell the stock sells at ₹380 with a premium of ₹20. The
time value of option will be
a) ₹10
b) ₹-10
c) ₹20
d) 0

Directorate of Studies, The Institute of Cost Accountants of India (Statutory Body under an Act of Parliament) Page 70
Work Book: Strategic Financial Management

(vi) The spot value of NIFTY is 6430. An investor bought a two month NIFTY for 6410 call option for a premium
of ` 24. The option is
a) In-the Money
b) At-the Money
c) Out-of the Money
d) Insufficient Data

(vii) Shares of C Ltd. is traded at ₹1150. An investor is bullish about the market. He buys two one month call
option contracts (one contract is 100 shares) on C Ltd. with a strike price of ₹1195 at a premium of ₹35
per share. Three months later, if the share is selling at ₹1240 what will be net profit/loss of the investor on
the position?
a) ₹1000
b) ₹1200
c) ₹1500
d) ₹2000

(viii) A stock index currently stands at 7000. The risk free interest rate is 8% p.a. continuously compounded
and the dividend yield on the index is 4% p.a. What should be the futures price for a four month
contract? [Given e (.08-.04)4/12= 1.013423]
a) 7093.96
b) 7097.34
c) 7098.68
d) 7099.25

Solution:

(i) a (iii) b (v) d (vii) d


(ii) c (iv) b (vi) a (viii) a

2. An investor has bought a futures contract on the stock of Maruti Udyog Ltd. at ₹410. Each contract consists of
400 shares. The initial margin is set by the exchange at 5%, while the maintenance margin is 90% of the
initial margin. Clearing prices of the stock for next 10 days are given below:

Day 1 2 3 4 5 6 7 8 9 10
Price (₹) 410 420 400 390 440 441 450 460 455 465

Assume that on the 10th day, the investor squares off his position at ₹465. Find out the gain and losses of long
and short positions of the investor. You are requested to show all necessary calculations.

Directorate of Studies, The Institute of Cost Accountants of India (Statutory Body under an Act of Parliament) Page 71
Work Book: Strategic Financial Management
Solution:

Day Clearing Profit/Loss for Day Margin Account Margin Call


price Long Position Short Long Position Short Position Long Position Short Position
Position
1. 410 - -
2. 420 10*400=4000 -4000 8200+4000=12200 8200-4000=4200 8200-4200=4000
3. 400 -20*400=-8000 8000 12200-8000=4200 8200+8000=16200 8200-4200=4000
4. 390 -10*400=-4000 4000 8200-4000=4200 16200+4000=20200 8200-4200=4000
5. 440 50*400=20000 -20000 8200+20000=28200 20200-20000=200 8200-200=8000
6. 441 1*400=400 -400 28200+400=28600 8200-400=7800
7. 450 9*400=3600 -3600 28600+3600=32200 7800-3600=4200 8200-4200=4000
8. 460 10*400=4000 -4000 32200+4000=36200 8200-4000=4200 8200-4200=4000
9. 455 -15*400=-6000 6000 36200-6000=30200 8200+6000=14200
10. 465 10*400=4000 -4000 30200+4000=34200 14200-4000=10200

Profit/Loss= Final Margin Position—Initial margin position-Margin call paid


Margin Call paid= Long Position=8000; Short position= 12000
Long Position profit/loss: =34200-8200-8000=18000
Short position profit/loss:=10200-8200-20000=28200-10200=18000
Minimum margin: 410*400*5%=8200
Maintenance margin: 6560*90%=7380

3. A portfolio manager owns three stocks and its details are under:
Stock Shares owned Stock Price (₹) Beta
X 4 Lakh 400 1.1
Y 8 Lakh 300 1.2
Z 12 lakh 100 1.3

The BSE-SENSEX is at 28000 and futures price is 28560. Use stock index futures to (i) decrease the portfolio
beta to 0.8 and (ii) increase the portfolio beta to 1.5. Assume the index factor is 100. Find out the number of
contacts to be bought or sold of stock index futures.

Solution:

Computation of existing portfolio beta


Stock Market value of stock (₹ in Lakh) Proportion Beta of the stock Weighted beta
X 1600 4/13 1.1 0.34
Y 2400 6/13 1.2 0.55
Z 1200 3/13 1.3 0.30
5200 1.19
Value per futures contract= Index price per contract*Lot size per futures contract = 28000*100 = ₹28,00,000

Directorate of Studies, The Institute of Cost Accountants of India (Statutory Body under an Act of Parliament) Page 72
Work Book: Strategic Financial Management
(i) To reduce portfolio beta to 0.8, the manager should sell index futures contract.
 Portfolio Value =₹5200 Lakh
 Value per futures contract= Index price per contract*Lot size per futures contract=28000*100= ₹28,00,000
 Beta of the existing portfolio=1.19
 Desired beta of the new portfolio =0.8
Beta of the Portfolio −Desired Value of Beta
 No of Contracts to be sold= Portfolio Value×
Value of the Futures Contrcat
(1.19−0.8)
No of Contracts= 5200 Lakh× = 72 contracts
28 Lakh

(ii) To increase the portfolio beta to 1.5 the manager should buy index futures contract.
 Portfolio Value =₹5200 Lakh
 Value per futures contract= Index price per contract*Lot size per futures contract
= 28000*100= ₹28,00,000= ₹28 Lakh
 Beta of the existing portfolio=1.19
 Desired beta of the new portfolio =1.5
Desired Value of Beta −Beta of the Portfolio
 No of Contracts to be sold= Portfolio Value×
Value of the Futures Contrcat
(1.5−1.19)
No of Contracts to be bought= 5200 Lakh× = 57.57=58 contracts
28 Lakh

4. XYZ Ltd. shares are presently quoted at `100. The 3 Month Call Option carries a premium of `15 for an
Exercise Price of `120 and a 3 Month’s put option carries a premium of `20 for a strike price `120.
If the spot price on the expiry date is in the range of `90 to `160 with an interval of `5, calculate Net Pay-Off
along with graph for both call option and put option from the option buyer’s perspective and option writer’s
perspective.

Solution:
Calculation of Net Payoff of Call Option Buyer and Writer
Call Option
Spot Price Exercise Price (`) Gross Payoff Premium Action Net Payoff (Long/Buyer) Net Payoff (Short/Seller)
90 120 0 15 Lapse (15) 15
95 120 0 15 Lapse (15) 15
100 120 0 15 Lapse (15) 15
105 120 0 15 Lapse (15) 15
110 120 0 15 Lapse (15) 15
115 120 0 15 Lapse (15) 15
120 120 0 15 Lapse (15) 15
125 120 5 15 Exercise (10) 10
130 120 10 15 Exercise (5) 5
135 120 15 15 Exercise 0 0
140 120 20 15 Exercise 5 (5)
145 120 25 15 Exercise 10 (10)
150 120 30 15 Exercise 15 (15)
155 120 35 15 Exercise 20 (20)
160 120 40 15 Exercise 25 (25)

Directorate of Studies, The Institute of Cost Accountants of India (Statutory Body under an Act of Parliament) Page 73
Work Book: Strategic Financial Management

Put Option
Spot Price Exercise Price (`) Gross Payoff Premium Action Net Payoff (Long/Buyer) Net Payoff (Short/Seller)
90 120 30 20 Exercise 10 (10)
95 120 25 20 Exercise 5 (5)
100 120 20 20 Exercise 0 0
105 120 15 20 Exercise (5) 5
110 120 10 20 Exercise (10) 10
115 120 5 20 Exercise (15) 15
120 120 0 20 Lapse (20) 20
125 120 (5) 20 Lapse (20) 20
130 120 (10) 20 Lapse (20) 20
135 120 (15) 20 Lapse (20) 20
140 120 (20) 20 Lapse (20) 20
145 120 (25) 20 Lapse (20) 20
150 120 (30) 20 Lapse (20) 20
155 120 (35) 20 Lapse (20) 20
160 120 (40) 20 Lapse (20) 20

5. The following data relates to share price of A Ltd.:


Current price per share ₹1,800
6 months future's price/share ₹1,950
Assuming it is possible to borrow money in the market for transactions in securities at 12% per annum, you
are required:
(i) to calculate the theoretical minimum price of a 6-months forward purchase; and
(ii) to explain arbitrate opportunity.

Answer:

(i) Calculation of theoretical minimum price of a 6 months forward contract-


Theoretical minimum price = ₹1,800 + (₹1,800 x 12/100 x 6/12) = ₹1,908

(ii) Arbitrage Opportunity-


The arbitrageur can borrow money @ 12 % for 6 months and buy the shares at ₹ 1,800.
At the same time he can sell the shares in the futures market at ₹1,950. On the expiry date 6 months later, he
could deliver the share and collect ₹1,950 pay off ₹1,908 and record a profit of ₹ 42 (₹1,950 – ₹1,908)

6. Consider a two year American call option with a strike price of ₹ 100 on a stock the current price of which is
also ₹ 100. Assume that there are two time periods of one year and in each year the stock price can move
up or down by equal percentage of 20%. The risk free interest rate is 6%. Using binominal option model,
calculate the probability of price moving up and down. Also draw a two step binomial tree showing prices
and payoffs at each node.

Directorate of Studies, The Institute of Cost Accountants of India (Statutory Body under an Act of Parliament) Page 74
Work Book: Strategic Financial Management
Solution:

Stock prices of two-stage binomial model

D (144)

B (120)

A (100) E(96)

C (80)

F(64)

Using the single period model, the probability of price increase is


1+r −d 1.06−0.80 0.260
P= = = =0.65
u−d 1.20−0.80 0.40
P=Probability
r=Risk free interest rate =6%=0.06
d=Downward movement i.e.20% =1-0.20=0.80
u=Upward movement i.e.=1+0.20=1.20

Therefore the p of price decrease = 1-0.65 = 0.35


The two step Binominal tree showing price and pay off

D (144) Payoff (44)

B (120)

A (100) E(96) Payoff (0)

C (80)

F(64) Payoff (0)

The value of an American call option at nodes D, E and F will be equal to the value of European option at these
nodes and accordingly the call values at nodes D, E and F will be 44, 0 and 0 using the single period binomial
model the value of call option at node B is

Payoff of at Node D∗ 1−p + Payoff at Node E∗p 44∗0.65+0∗0.35 0.260


C= = = =26.98
1+r 1+0.06 0.40

At node B the payoff from early exercise will pay ` 10, which is less than the value calculated using the single
period binomial model. Hence at node B, early exercise is not preferable and the value of American option at

Directorate of Studies, The Institute of Cost Accountants of India (Statutory Body under an Act of Parliament) Page 75
Work Book: Strategic Financial Management
this node will be ` 13.49. If the value of an early exercise had been higher it would have been taken as the
value of option. The value of option at node ‗A‘ is
26.98∗0.65+0∗0.35 0.260
= =25.45
1+0.06 0.40

7. From the following data for certain stock, find the value of a call option:
Price of stock now = ₹80
Exercise price = ₹75
Standard deviation of continuously compounded annual return = 0.40
Maturity period = 6 months
Annual interest rate = 12%
[Given 𝐞.𝟎𝟔 =1.062; In 1.0667 = 0.0646; N (0.5820) = 0.7197; N(0.2992) = 0.6176]

Solution :

Applying the Black Scholes Formula,


Value of the Call option now:
The Formula C = SN(d )- Ke(-rt) N(d2 )

Where,
C = Theoretical call premium
S = Current stock price
t = time until option expiration
K = option striking price
r = risk-free interest rate
N = Cumulative standard normal distribution
e = exponential term
σ = Standard deviation of continuously compounded annual return.
In = natural logarithm

= 0.5820
d2 = 0.5820 – 0.2828 = 0.2992
N(d1) = N (0.5820)
N(d2) = N (0.2992)
Price = C = SN(d1 )- Ke(-rt) N(d2 )
= 80 x N(d1) – (75/1.062) x N(d2)

Directorate of Studies, The Institute of Cost Accountants of India (Statutory Body under an Act of Parliament) Page 76
Work Book: Strategic Financial Management

Value of option

N(d1) = N (0.5820) = 0.7197


N(d2) = N(0.2992) = 0.6176

= 57.57 – 70.62 x 0.6176


= 57.57 – 43.61
= ₹13.96

8. Following information is available for two firms.


Firm Objective Fixed Rate Floating Rate
A Floating Rate 10% LIBOR+0.75%
B Fixed Rate 11% LIBOR+1.00%

Explain how the two firms would enter into a swap transaction to reduce their interest costs, if Firm A does
not want to pay more than LIBOR+0.35%.

Solution:

This is the interest rate swap without a bank intermediary.


Swap Design:
Step 1: Find the interest differential in fixed market. Here it is 100 bps.
Step 2: Find the interest differential in floating market. Here it is 25 bps.
Step 3: The quality spread is 75 bps (100-25) bps
Step 4: Firm‘s A objective is to go for floating payment but wants to pay LIBOR+0.35%. In other words, as against
the market cost of LIBOR+0.75%, it wants to pay LIBOR+0.35% implying a savings of 0.40%. Out of possible 0.75%
(quality spread), B would save 0.35% i.e. B‘s cost would be 10.65%.

9. Company P Ltd. and Q Ltd. have been offered the following rate per annum on a ₹200 crore five year loan:
Company Fixed Rate Floating Rate
P Ltd. 12.0% MIBOR+0.1%
Q Ltd. 13.4% MIBOR +0.6%

Company P Ltd. requires a floating rate loan and Q Ltd. requires a fixed rate loan.

You are required to design a swap arrangement that will net a bank acting as intermediary at 0.5% p.a. and
be equally attractive to both the companies. Also find out the effective interest rates.

Directorate of Studies, The Institute of Cost Accountants of India (Statutory Body under an Act of Parliament) Page 77
Work Book: Strategic Financial Management
Solution:

Particulars `
(a) Difference in Floating Rates [(MIBOR + 0.1%) - (MIBOR + 0.6%)] 0.5%
(b) Difference in Fixed Rates [13.4% - 12%] 1.4%
(c) Net Difference {[(a) - (b)] in Absolute Terms} 0.9%
(d) Amount paid for arrangement of Swap Option (0.5%)
(e) n et Gain [(c) - (d)] 0.4%
(f) Company PQR‘s share of Gain [0.4% X 50%] 0.2%
(g) Company DEF‘s share of Gain [0.4% X 50%] 0.2%

P Ltd. is the stronger Company (due to comparative interest advantage). P has an advantage of 1.40% in
Fixed Rate and 0.50% in Floating Rate. Therefore, P Ltd. enjoys a higher advantage in Fixed Rate loans.
Therefore, P Ltd. will opt for Fixed Rate Loans with its Bankers. Correspondingly Q Ltd. will opt for Floating Rate
Loans with its bankers.

P Ltd. Q Ltd.
1. P Ltd. will borrow at fixed rate. 1. Q Ltd. will borrow at Floating rate.
2. Pay interest to bankers at fixed rate (i.e. 2. Pay interest to bankers at Floating rate (MIBOR+0.6%)
12%) 3. Will pay Company P Ltd. interest amount differential
3. Will collect from Company Q Ltd. interest i.e. Interest computed at fixed rate (12%) Less Interest
amount differential i.e. Interest computed Computed at Floating Rate of (MIBOR+0.1%)=11.9%-
at fixed rate (12%) Less Interest Computed MIBOR
at Floating Rate of (MIBOR+0.1%)=11.9%- 4. Pay to Company P Ltd. its share of gain (0.2%).
MIBOR 5. Pay Commission Charges to the bank for arranging
4. Receive share of gain from Company Q interest rate swap i.e. 0.5%.
Ltd. (0.2%). 6. Effective Interest Rate= (2+3+4+5)= MIBOR + 0.60 % +
5. Effective Interest Rate= 2-3=12%-(11.9%- 11.9% - LIBOR + 0.5% + 0.2%=13.20%
MIBOR)-0.2%=MIBOR-0.1%

10. X Ltd. and Y Ltd. both wish to raise USD 20 million loan for 5 years. X Ltd. has the choice of issuing fixed rate
debt at 7.50% of floating rate debt at LIBOR+25bps. On the other, Y Ltd. which has a lower credit rating, can
issue fixed rate debt of the same maturity at 8.45% or floating rate at LIBOR+37bps. X Ltd. prefers to issue
floating rate debt and Y Ltd. prefers fixed rate debt with a lower coupon. City bank is in the process of
arranging an interest rate swap between these two companies.
X Ltd. negotiates to pay the bank a floating rate of LIBOR while the bank agrees to pay X Ltd. a fixed rate of
7.60%.
Y Ltd. agrees to pay the bank a fixed rate of 7.75% while the bank pays Y Ltd. a floating rate of LIBOR flat.
You are required to:
a) With a schematic diagram, show how the swap deal can be structured.
b) What are interest savings by each company?
c) How much would City Bank receive?

Directorate of Studies, The Institute of Cost Accountants of India (Statutory Body under an Act of Parliament) Page 78
Work Book: Strategic Financial Management

Solution:

Calculation of Quality Spread Differential


Company Objective Fixed Rate Floating Rate
X Ltd. Floating Rate 7.5% p.a. LIBOR+0.25%
Y Ltd. Fixed Rate 8.45% LIBOR+0.37%
Difference in risk premium 0.95% 0.12%
Net Differential 0.83%

The differential between two markets =0.83%


This needs to shared between X Ltd. and Y Ltd. and City Bank

7.50% 7.6% LIBOR


X Ltd. City Bank
Y Ltd.
LIBOR+0.37%
LIBOR 7.75%

Economics of Swap Deal


X Ltd. Bank Y Ltd.
Paid to lender (7.50%) (LIBOR+0.37%)
Bank pays to X Ltd. 7.60% (7.60% ) -
Y Ltd. pays Bank - 7.75% (7.75%)
X Ltd. pays Bank (LIBOR) LIBOR -
Bank Pays Y Ltd. (LIBOR) LIBOR
Net Position (LIBOR-0.10%) 0.15% (8.12%)
Cost without Swap LIBOR+0.25% - 8.45%
Gain 0.35% 0.15% 0.33%

(ii) Savings
X Ltd.: [LIBOR+0.25%+7.60%-7.50%-LIBOR)= 0.35%
Y Ltd.: [8.45%+LIBOR-7.75%-LIBOR-0.37%]= 0.35%

(iii) Gain to City Bank


LIBOR-LIBOR+7.75%-7.60%=0.15%

Directorate of Studies, The Institute of Cost Accountants of India (Statutory Body under an Act of Parliament) Page 79
Work Book: Strategic Financial Management

Study Note – 11

Financial Risk Management in International Operations

Q.1 Choose the correct alternative:

(i) Arbitrage pricing theory model helps to


a) Reduce risk
b) Eliminate arbitrage
c) Identify the equilibrium asset price
d) None of the above

(ii) In July, the one year interest rate is 4% on Swiss Francs and 13% on US dollars. If the current exchange rate
SFr 1=$0.63, what is the expected future exchange rate in one year?
a) $ 0.5561
b) $ 0.6845
c) $ 0.8542
d) $ 0.8283

(iii) Between 2000 and 2015, the ¥/$ exchange rate moved from ¥226.63 to ¥93.96. During this same 15 year
period, the consumer price index (CPI) in Japan rose from 91.0 to 119.2 and the US CPI rose from 82.4 to
152.4. If PPP held over this period, what would the ¥/$ exchange rate have been in 2015?
a) ¥ 140.13
b) ¥ 152.15
c) ¥160.51
d) ¥ 180.18

(iv) The 90-day interest rate is 1.85% in USA and 1.35% in the UK and the current spot exchange rate is $1.6/£.
The 90-day forward rate is-
a) $1.607893
b) $ 1.901221
c) $ 1.342132
d) $ 1.652312

(v) The current spot rate for the U.S. dollar is ₹ 66. The expected inflation rate is 6.5% in India and 3% in USA. The
expected rate of dollar a year hence is
a) ₹72.33
b) ₹72.12
c) ₹69.33
d) ₹66.89

Directorate of Studies, The Institute of Cost Accountants of India (Statutory Body under an Act of Parliament) Page 80
Work Book: Strategic Financial Management
(vi) The following rates are prevailing: Euro/$:1.1916/1.1925 and $/£:1.42/1.47 what will be the cross rate?
a) 1.6921/1.7530
b) 1.7530/1.6921
c) 1.6921/1.1925
d) 1.7530/1.1916

Solution:

(i) c (iii) c (v) d


(ii) b (iv) a (vi) a

2. Identify and briefly discuss the determinants of foreign exchange rates.

Answer:

The determinants of foreign exchange rates are as follows:

(i) Interest Rate Differentials: Higher rate of interest for a investment in a particular currency can push up the
demand for that currency, which will increase the exchange rate in favour of that currency.
(ii) Inflation Rate Differentials: Different countries‘ have differing inflation rates, and as a result, purchasing
power of one currency will depreciate faster than currency of some other country. This contributes to
movement in exchange rate.
(iii) Government Policies: Government may impose restriction on currency transactions. Through RBI, the
Government, may also buy or sell currencies in huge quantity to adjust the prevailing exchange rates.
(iv) Market Expectations: Expectations on changes in Government, changes in taxation policies, foreign trade,
inflation, etc. contributes to demand for foreign currencies, thereby affecting the exchange rates.
(v) Investment Opportunities: Increase in investment opportunities in one country leads to influx of foreign
currency funds to that country. Such huge inflow will amount to huge supply of that currency, thereby
bringing down the exchange rate.
(vi) Speculations: Speculators and Treasury Managers influence movement in exchange rates by buying and
selling foreign currencies with expectations of gains by exploiting market inefficiencies. The quantum of their
operations affects the exchange rates.

3. Discuss the process for raising Equity through ADR.

Answer:

The processes for raising Equity through ADR are as follows:


(a) Issue Intermediaries: ADRs are issued by Overseas Depository Bank (ODB), who has a Domestic Custodian
Bank (DCB) in India.
(b) Deposit of Securities: Company willing to raise equity through ADRs should deposit the securities with the
DCB in India.

Directorate of Studies, The Institute of Cost Accountants of India (Statutory Body under an Act of Parliament) Page 81
Work Book: Strategic Financial Management
(c) Authorization for Issue of ADRs: The Indian Company authorizes the ODB to issue ADR against the security of
Company‘s Equity Shares.
(d) Issue of ADR: ODB issues ADRs to investors at a predetermined ratio to the Company‘s securities.
(e) Redemption of ADR: When an investor redeems his ADRs, the appropriate number of underlying equity
shares or bonds is released.
(f) Dividend / Interest: The Indian Company pays interest to the ODB, which in turn distributes dividends to the
ADR holders based on the prevailing exchange rate.

4. Who Can Invest in P-Notes?

Answer:

The following persons/entities are eligible to invest in Participatory Notes (P-Notes):


(a) Any entity incorporated in a jurisdiction that requires filing of constitutional and/or other documents with a
registrar of companies or comparable regulatory agency or body under the applicable companies
legislation in that jurisdiction;
(b) Any entity that is regulated, authorized or supervised by a central bank, such as the Bank of England, the
Federal Reserve, the Hong Kong Monetary Authority, the Monetary Authority of Singapore or any other
similar body provided that the entity must not only be authorized but also be regulated by the aforesaid
regulatory bodies;
(c) Any entity that is regulated, authorized or supervised by securities or futures commission, such as the
Financial Services Authority (UK), the Securities and Exchange Commission, the Commodities Futures Trading
Commission, the Securities and Futures Commission (Hong Kong or Taiwan), Australia Securities and
Investments Commission (Australia) or other securities or futures authority or commission in any country, state
or territory;
(d) Any entity that is a member of securities or futures exchanges such as the New York Stock Exchange
(Subaccount), London Stock Exchange (UK), Tokyo Stock Exchange (Japan), NASD (Sub-account) or other
similar self-regulatory securities or futures authority or commission within any country, state or territory
provided that the aforesaid organizations which are in the nature of self regulatory organizations are
ultimately accountable to the respective securities / financial market regulators.
(e) Any individual or entity (such as fund, trust, collective investment scheme, Investment Company or limited
partnership) whose investment advisory function is managed by an entity satisfying the criteria of (a), (b), (c)
or (d) above.

5. Identify the issues relating to International Capital Budgeting.

Answer:

The decision to invest abroad takes a concrete shape when a future project is evaluated in order to ascertain
whether the implementation of the project is going to add to the value of the investing company. The
evaluation of the long term investment project is known as capital budgeting. The technique of capital

Directorate of Studies, The Institute of Cost Accountants of India (Statutory Body under an Act of Parliament) Page 82
Work Book: Strategic Financial Management
budgeting is almost similar between a domestic company and an international company. However, the one
has to address the following issues related to International Capital Budgeting:
a) Exchange rate fluctuations capital market segmentation,
b) International financing arrangement of capital and related to cost of capital,
c) International taxation,
d) Country risk or political risk etc.

6. You are given the following information–


$/£ 1.3690/1.3728
S.Fr/DEM 1.0050/1.0098
$/S.Fr 0.8810 / 0.8823
And if DEM / £ in the market are 1.5580 /1.5596

You are required to find out any arbitrage opportunity exists.


If so, show how $20,000 available with you can be used to generate risk - less profit.

Solution:

Calculation of Cross Rate


(a) Bid [DEM / £] = Bid [$ / £] x Bid [S Fr. / $] x Bid [DEM / S Fr.]
= Bid [$ / £] x 1 / Ask [$ / S Fr.] x 1 / Ask [S Fr. / DEM)
=1.3690x 1 / 0.8823x 1 / 1.0098
=1.55902

(b) Ask [DEM/£] = Ask [$ / £] X Ask [S Fr./$] X Ask [DEM / S Fr.]


= Ask [$ / £] X 1/ Bid [$ / S Fr.] X 1 / Bid [S Fr. / DEM]
= 1.3728x 1/0.8810 x 1/1.0050
=1.55048

Cross Rate Market Rate


DEM / £ 1.55902-1.55048 1.5580 /1.5596

Since both the rates are apart there exist an arbitrage opportunity.
Nature of Quote Buying Foreign Currency Buying Foreign Currency
(Converting Home Currency into (Converting Home Currency into
Foreign Currency) Foreign Currency)
Direct Quote, relevant rate is Ask Rate Bid Rate
Indirect Quote, relevant rate is 1 ÷ Bid Rate 1 ÷ Ask Rate

Directorate of Studies, The Institute of Cost Accountants of India (Statutory Body under an Act of Parliament) Page 83
Work Book: Strategic Financial Management

Sell US $20,000 @1.3728 Gain of US $ 97.04 Sell S. Fr 22811.63@ 0.8810


(US $20,000 ÷ 1.3728) (22811.63x 0.8810)
Receive £145687.65 US $ (20097.04- 20,000) Receive US $20097.04

Sell £ at the available Sell DEM 22698.14 @1.0050


DEM / £ 1.5580 (DEM 11,351.04 x 1.005)
(£145687.65x 1.5580) Receive S.Fr. 22811.63
Receive DEM 22,698.14

7. S Ltd. an Indian based company has subsidiaries in US and UK whose forecast surplus fund for the next 30
days (June 2018) are given below:
US subsidiary: $ 12.00 million
UK subsidiary: £ 6.00 million

The following information pertaining to exchange rates are obtained:


$/₹ £/₹
Spot 0.0243 0.0148
30 days forward 0.0245 0.0150

The borrowing/deposit rates per annum (simple) are available:


₹ 8.4%/7.5%
$ 1.6%/1.5%
£ 4.0%/3.8%

The Indian operation is forecasting a cash deficit of ₹ 400 million. It is assumed that interest rates based on
over a year of 360 days.

Required:
i) Calculate the cash balance in Rupees at the end of 30 days period (at the end of June 2018) for each
company under each of the following scenarios ignoring transaction costs and taxes:
a) Each company invests/finance its own cash balance/deficits in local currency independently.
b) Cash balances are pooled immediately in India and the net balances are invested/borrowed for the
30 days period.
ii) Which method do you think preferable from the parent company’s (S Ltd.) point of view?

Solution:

Directorate of Studies, The Institute of Cost Accountants of India (Statutory Body under an Act of Parliament) Page 84
Work Book: Strategic Financial Management

a) Computation of Cash Balances at the end of 30 days of S Ltd. (At the end of June 2018)

i) Acting Independently
(Figures in million)
Particulars India US Subsidiary UK Subsidiary
Surplus/Deficit (₹ 400) $ 12.00 £ 6.00
Interest on Investment 7.50% 1.50% 3.80%
Interest on borrowing 8.40% 1.60% 4.00%
Interest 400×(0.084/12)=2.80 12×(0.015/12)=0.015 6×(0.038/12)=0.019
Values after adjusting interest (₹ 402.80) ($ 12.015) (£ 6.019)
Values in Rupee term (using (₹ 402.80) (₹ 490.408) (₹ 491.267)
forward rate) (12.015/0.0245) (6.019/0..150)
Net value in Rupees (Balance):
= ₹ (402.80)+490.408+491.267
= ₹ 488.875 million

ii) Cash balance are pooled immediately


(Figure in million)
Particulars ₹
India (400.00)
US Subsidiary (12.00/0.0243) [Spot rate] 493.827
UK Subsidiary (6.00/0.0148) [Spot rate] 405.405
Immediate cash balance 499.232
Interest for 30 days [499.232×(0.075/12)] 3.120
Cash balance at the end of 30 days 502.352

b) Decision from S Ltd.’s point of view:


From S Ltd.‘s point of view ‗immediate cash pooling to India‘ is preferable as it maximizes the total cash
balance of the company after 30 days comparing to acting independently.

8. On 19th April 2018 the following are the spot rates:


Spot EURO/USD 1.20000 USD/INR 44.8000

Following are the quotes of European Options:


Currency Pair Call/Put Strike Price Premium Expiry Date
EURO/USD Call 1.2000 $ 0.035 July, 19
EURO/USD Put 1.2000 $ 0.04 July 19
USD/INR Call 44.8000 ₹ 0.12 Sep 19
USD/INR Put 44.8000 ₹0.04 Sep 19

Directorate of Studies, The Institute of Cost Accountants of India (Statutory Body under an Act of Parliament) Page 85
Work Book: Strategic Financial Management
i) A trader sells an at-the-money spot straddle expiring at three months (July 19). Calculate gain or loss if
three months later the spot rate is EURO/USD 1.2900.
ii) Which strategy gives a profit to the dealer if five months later (Sep. 19) expected spot rate is USD/INR
45.00. Also calculate profit for a transaction USD 1.5 million.

Solution:

i) Straddle is a portfolio of a Call and a Put option with identical Strike price. A trader sells Straddle of at the
Money Straddle by selling a call option and put option with strike price of USD per EURO.
He will receive premium of $ 0.035+$0.040=$ 0.075.

At the expiry of three months spot rate is 1.2900 i.e. higher than Strike Price. Hence, buyers of the call option
will exercise the option, but buyer of Put option will allow the option to lapse.

Profit or loss to a trader is:


Premium received $0.075
Loss on call option exercised (1.2900-1.200) $0.090
So the Net loss is $(0.075-0.090)= $0.015 per EURO.

ii) BUY Strategy i.e. either Call or Put:


When price is expected to go up then call option is beneficial,
On 19th April to pay premium 15,00,000 @₹0.12 i.e. ₹ 1,80,000
On 19th September exercise call option to gain 15,00,000 @₹0.20 ₹ 3,00,000
Net Gain or Profit ₹ 1,20,000

9. X Ltd. an Indian company has a payable of US$ 1,00,000 due in 3 months. The company is considering to
cover the payable through the following alternatives:
i) Forward contract,
ii) Money market
iii) Option

The following information is available with the company:


Exchange rate:
Spot ₹/$45.50/45.55
3-m Forward 45.90/46.00

Interest Rate (%): Per annum


US 4.5/5.0 (Deposit/Borrow)
India 10.0/11.0 (Deposit/Borrow)

Call option on $ with a strike price of ₹46is available at a premium of ₹ 0.10/$. Put option on $ with a strike
price of ₹ 46.00 is available with a premium of ₹ 0.05/$.
Treasury department of the company forecasted the future spot rate after 3 months to be:
Spot rate after 3-mProbability

Directorate of Studies, The Institute of Cost Accountants of India (Statutory Body under an Act of Parliament) Page 86
Work Book: Strategic Financial Management
₹ 45.60/$ 0.10
₹ 46.00/$ 0.60
₹ 46.40/$ 0.30

You are required to suggest the best alternative of hedging.

Solution:

Exchange rate: ₹/$


Spot: 45.50/45.55
3-m forward: 45.90/46.00

3-m interest rate (%)


US 4.5/5.0
India 10.0/11.0

i. Forward Hedge:
After 3-m, outflow of ₹ for the month is ₹ (1,00,000×46.00)=46,00,000

ii. Forward Hedge


The firm should borrow ₹ and convert it into $ at the spot rate. Then the $ proceeds for 3 –m to be invested
and the payable will be settled at maturity out of the $ investment.

1,00,000
$ to be invested to get $ 1,00,000 3-m hence is: 0.045 = $98,887.52
1+
4

To get $ 98,887.52 the amount of₹ required is = (98,887.52×45.55)= ₹ 45,04,326.54. So, the firm has to borrow a
sum of ₹ 45,04,326.54.
0.11
Hence, rupee repayment after 3-m is=₹ 45,04,326.54×1 + =₹46,28,195.52
4

iii. Option Hedge


Since the firm has a $ liability, it should go long on call $ option. That means the firm will buy $ call option
with a strike price of ₹ 46.00 at a premium of ₹ 0.10/$.

So, total premium paid is ₹ (1,00,000×0.10)= ₹ 10,000.


Possible spot rate after 3-m (₹/$) Whether to exercise Option Total ₹ outflow Probability
45.60 No 45,70,000 0.10
46.00 No 46,10,000 0.60
46.40 Yes 46,00,000 0.30

Expected rupee outflow after 3 month is = ₹(45,70,000×0.10)+(46,10,000×0.60)+ (46,10,000×0.30)= ₹46,06,000.


The firm can also go short on the put option, that is sell $ put option with a strike price of ₹46.00 at a
premium of ₹0.05/$.

Total premium received is ₹ (1,00,000×0.05)= ₹ 5,000


Directorate of Studies, The Institute of Cost Accountants of India (Statutory Body under an Act of Parliament) Page 87
Work Book: Strategic Financial Management
Possible spot rate after 3-m (₹/$) Whether to exercise Option Total ₹ outflow Probability
45.60 No 45,95,000 0.10
46.00 No 45,95,000 0.60
46.40 Yes 46,35,000 0.30

Expected rupee outflow after 3 month is = ₹(45,95,000×0.10)+(45,95,000×0.60)+ (46,35,000×0.30)= ₹ 46,07,000

Suggestion: ‗Forward Hedge‘ is suggested for X Ltd. to cover the payable since the rupee outflow is less
than the outflow under money market hedge and also less than the expected outflow under option covers.

Directorate of Studies, The Institute of Cost Accountants of India (Statutory Body under an Act of Parliament) Page 88

You might also like